You are on page 1of 68

Total Marks : 200

TEST - 23 (MOCK TEST - 18) Mark Scored : 0

1 Consider the following about Molecular machines.


1. They convert chemical energy into mechanical forces and motion.
2. Ribosome is an example of a molecular machine.
3. Molecular machines can be put to work as tiny motors rotating objects far bigger than their size.

Select the correct answer using the codes below.


A. 1 only
B. 1 and 3 only
C. 2 and 3 only
D. 1, 2 and 3

Your Answer :
Correct Answer : D

Answer Justification :

Concept: They are the world’s smallest machines. Their working is inspired by proteins that
naturally act as biological machines within cells.

Molecular machines are discrete number of synthetic molecular components fused together.

They produce quasi-mechanical movements in response to specific external stimuli such as


light or temperature change.

They are either natural or synthetic molecules that convert chemical energy into mechanical
forces and motion.

Justification: Statement 2: Ribosomes are the protein builders or the protein synthesizers of the
cell. They are like construction guys who connect one amino acid at a time and build long chains.
Ribosomes are special because they are found in both prokaryotes and eukaryotes.

Another example of a biological motor is the protein kinesin, which uses the hydrolysis of adenosine
triphosphate to move along microtubule filaments.

2016 Nobel Prize in Chemistry was based on the individual efforts in developing molecular
machines.

Learning: Molecular machines can be put to work as tiny motors, pistons ratchets or wheels to
produce mechanical motion and can move objects many time their size.

Future Potential Applications include their functions as artificial muscles to power tiny robots or
even prosthetic limbs in case of Bionics, new sensors, materials and energy storage systems, deliver
drugs within the human body directly to target a specific area of tissue etc.

Q Source: https://www.nature.com/subjects/molecular-machines-and-motors

www.insightsias.com 1
Total Marks : 200
TEST - 23 (MOCK TEST - 18) Mark Scored : 0

2 The Market Stabilization Scheme (MSS) was launched with the objective of strengthening RBI's ability
to
1. Control liquidity in the market
2. Regulate government borrowing

Which of the above is/are correct?


A. 1 only
B. 2 only
C. Both 1 and 2
D. None

Your Answer :
Correct Answer : A

Answer Justification :

Background: Historically, the RBI had been sterilizing the effects of significant capital inflows on
domestic liquidity by offloading parts of the stock of Government Securities held by it.

The MSS was devised since continuous resort to sterilization by the RBI depleted its limited stock of
Government Securities and impaired the scope for similar interventions in the future.

Justification: Statement 1: Under this scheme, the GoI borrows from the RBI (such borrowing
being additional to its normal borrowing requirements) and issues Treasury-Bills/Dated Securities
that are utilized for absorbing excess liquidity from the market.

Therefore, the MSS constitutes an arrangement aiding in liquidity absorption, in keeping with the
overall monetary policy stance of the RBI, alongside tools like the Liquidity Adjustment Facility
(LAF) and Open Market Operations (OMO).

The securities issued under MSS, termed as Market Stabilization Scheme (MSS) Securities/Bonds,
are issued by way of auctions conducted by the RBI and are done according to a specified ceiling
mutually agreed upon by the GoI and the RBI.

Statement 2: It was not launched with the objective of regulating or controlling government
borrowing. It is just that the government securities are involved in the operation for offloading
liquidity from the market.

Q Source: Revision: Chapter on External Sector of India: Indian Economy: Ramesh Singh

3 Consider the following statements.


1. A person to be elected to the legislative assembly must be an elector for an assembly constituency
in the concerned state.
2. A person to be elected to the legislative council need not be an elector for assembly constituency in
the concerned state.

Which of the above is/are correct?


A. 1 only

www.insightsias.com 2
Total Marks : 200
TEST - 23 (MOCK TEST - 18) Mark Scored : 0

B. 2 only
C. Both 1 and 2
D. None

Your Answer :
Correct Answer : A

Answer Justification :

Justification: He must be an elector for assembly constituency in the concerned state both for the
council and the assembly.

This is also true for qualifying for Governor’s nomination in the state.

Moreover, he must be a member of a scheduled caste or scheduled tribe if he wants to contest a


seat reserved for them.

Q Source: Chapter 29: State Legislature: Indian Polity: M Laxmikanth

4 Consider the following statements about the Kheda Satyagraha of 1918.


1. It was the first Satyagraha movement launched in India by Gandhi
2. The peasants of Kheda were unable to pay high taxes of the British due to crop failure and plague
epidemic which inspired the Satyagraha.
3. During this time, Sardar Patel organised a major tax revolt within the Kheda community.

Select the correct answer using the codes below.


A. 1 and 2 only
B. 2 and 3 only
C. 1 and 3 only
D. 2 only

Your Answer :
Correct Answer : B

Answer Justification :

Justification: S1: It was the third Satyagraha movement after Champaran Satyagraha and
Ahmedabad mill strike. Gandhi ji organise this movement to support peasants of kheda district.

The peasants of Kheda could not be able to pay high taxes of British due to crop failure and plague
epidemic.

In Gujarat, Gandhi was chiefly the spiritual head of the struggle. His chief lieutenant, Sardar
Vallabhbhai Patel and a close coterie of devoted Gandhians, toured the countryside, organised
the villagers and gave them political leadership and direction.

The tax withheld, the government's collectors and inspectors sent in thugs to seize property

www.insightsias.com 3
Total Marks : 200
TEST - 23 (MOCK TEST - 18) Mark Scored : 0

and cattle, while the police forfeited the lands and all agrarian property. The farmers did not
resist arrest, nor retaliate to the force employed with violence.

The Government finally sought to foster an honourable agreement for both parties. The tax for the
year in question, and the next would be suspended, and the increase in rate reduced, while all
confiscated property would be returned.

Q Source: Chapter 12: Bipin Chandra: India’s Struggle for Independence

5 Consider the following statements. Coral reefs


1. Break the power of the waves during storms, cyclones and tsunamis
2. Form the spawning and nursery grounds for several fish species
3. Are being used in the treatment of cancer

Select the correct answer using the codes below.


A. 1 only
B. 1 and 2 only
C. 2 and 3 only
D. 1, 2 and 3

Your Answer :
Correct Answer : D

Answer Justification :

Justification: Healthy coral reefs are among the most biologically diverse and economically
valuable ecosystems on earth, providing valuable and vital ecosystem services.

Coral ecosystems are a source of food for millions; protect coastlines from storms and erosion;
provide habitat, spawning and nursery grounds for economically important fish species; provide
jobs and income to local economies from fishing, recreation, and tourism; are a source of new
medicines, and are hotspots of marine biodiversity.

Statement 1: By helping to prevent coastal erosion, flooding, and loss of property on the shore, the
reefs save billions of dollars each year in terms of reduced insurance and reconstruction costs and
reduced need to build costly coastal defences.

Statement 2: They form the nurseries for about a quarter of the ocean's fish, and thus provide
revenue for local communities as well as national and international fishing fleets.

Statement 3: A new class of proteins capable of blocking the HIV virus from penetrating T-cells has
been discovered. The proteins have been found in a coral from Australia's northern coast. Coral
reefs are also being used in treatment of cancer.

Q Source: https://coralreef.noaa.gov/aboutcorals/values/

6 Among the following, which of the following can be done by the State Legislative Assembly, but not by the State Legislative Council?

www.insightsias.com 4
Total Marks : 200
TEST - 23 (MOCK TEST - 18) Mark Scored : 0

1. Consideration of the report of State Finance Commission


2. Discussing the State Annual Budget
3. Approval of ordinances issued by the governor of the State
4. Election of the representatives of the state in the Rajya Sabha
5. Ratification of a constitutional amendment bill relevant to the State

Select the correct answer using the codes below.


A. 1, 2, 4 and 5 only
B. 2, 3 and 4 only
C. 4 and 5 only
D. 1, 3 and 5 only

Your Answer :
Correct Answer : C

Answer Justification :

Justification: The powers and status of the State Legislative council are broadly equal to that of
the State Legislative assembly, except in some cases.

Statement 1 and 2: Both have equal powers in the consideration of the reports of the constitutional
bodies like State Finance Commission, state public service commission and Comptroller and Auditor
General of India.

The council can discuss the budget but cannot vote on the demands for grants (which is the
exclusive privilege of the assembly).

Statement 3: Similar to passing of a law, ordinances also need to be approved by both houses.

Statement 4: Only the assembly participates in the election of the Rajya Sabha MPs based on
proportional representation system (single transferrable votes).

Statement 5: The council has no effective say in the ratification of a constitutional amendment bill.
Only the assembly ratifies it.

Q Source: Chapter 29: State Legislature: Indian Polity: M Laxmikanth

7 Consider the following statements about the Securities Transaction Tax (STT).
1. It is a type of an indirect tax.
2. It is levied on purchase or sale of securities that are listed on the Indian stock exchanges.
3. Off-market share transactions are also covered under it.

Select the correct answer using the codes below.


A. 1 and 2 only
B. 2 and 3 only
C. 2 only
D. 1 and 3 only

www.insightsias.com 5
Total Marks : 200
TEST - 23 (MOCK TEST - 18) Mark Scored : 0

Your Answer :
Correct Answer : C

Answer Justification :

Justification: STT is a type of direct tax payable on the value of taxable securities transaction done
through a recognized stock exchange in the country.

The securities on which STT is applicable are shares, bonds, debentures, derivatives, units issued by
any collective investment scheme, equity based government rights or interests in securities and
equity mutual funds.

Off-market share transactions are not covered under STT.

For equity transactions that are delivery-based, STT for purchase and sale is 0.1% of turnover and
for intra-day transactions, STT for purchase is nil and sale is 0.025% of the turnover.

Q Source: Revision: Chapter on Tax Structure in India: Indian Economy: Ramesh Singh

https://economictimes.indiatimes.com/wealth/invest/5-smart-things-to-know-about-securities-transac
tion-tax/articleshow/59398574.cms

8 The Ghadar party was


1. A revolutionary organization
2. Aiming at securing India's independence from British rule
3. Associated with Rashbehari Bose
4. Was founded in Punjab

Select the correct answer using the codes below.


A. 1, 2 and 3 only
B. 1 and 4 only
C. 2, 3 and 4 only
D. 1, 2 and 4 only

Your Answer :
Correct Answer : A

Answer Justification :

Learning: The Ghadar Party was an organization founded by Punjabi Indians in the United States
and Canada with the aim of securing India's independence from British rule.

Key members included Lala Har Dayal, Sohan Singh Bhakna, Abdul Hafiz Mohamed Barakatullah,
Kartar Singh Sarabha, and Rashbehari Bose.

After the outbreak of World War I, Ghadar party members returned to Punjab to agitate for
rebellion alongside the Babbar Akali Movement.

In 1915 they conducted revolutionary activities in central Punjab and organised uprisings. Their

www.insightsias.com 6
Total Marks : 200
TEST - 23 (MOCK TEST - 18) Mark Scored : 0

presence shook the hold of the British empire and police surveillance in Punjabi villages increased
in an attempt to crush the rebellion.

Q Source: Chapter 12: Bipin Chandra: India’s Struggle for Independence

9 Consider the following statements.


1. Like the Speaker, the Deputy Speaker is also elected by the assembly itself from amongst its
members.
2. The Speaker nominates a panel of chairmen of eminent citizens from outside of Parliament to guide
the affairs of the house.

Which of the above is/are correct?


A. 1 only
B. 2 only
C. Both 1 and 2
D. None

Your Answer :
Correct Answer : A

Answer Justification :

Justification: Statement 1: He is elected after the election of the Speaker has taken place. The
Deputy Speaker performs the duties of the Speaker’s office when it is vacant. He also acts as the
Speaker when the latter is absent from the sitting of assembly. In both the cases, he has all the
powers of the Speaker.

Statement 2: The Speaker nominates from amongst the members a panel of chairmen.

Any one of them can preside over the assembly in the absence of the Speaker or the Deputy
Speaker.

Q Source: Chapter 29: State Legislature: Indian Polity: M Laxmikanth

10 Consider the following statements.


1. The conduct of all elections to the panchayats is vested in the Election Commission of India.
2. The state election commissioner is appointed by the President of India.
3. The State Election Commission is subordinate to the Election Commission of India and presents its
annual report to the Commission.

Select the correct answer using the codes below.


A. 1 and 2 only
B. 2 and 3 only
C. 2 only
D. 1 and 3 only

Your Answer :

www.insightsias.com 7
Total Marks : 200
TEST - 23 (MOCK TEST - 18) Mark Scored : 0

Correct Answer : C

Answer Justification :

Justification: The superintendence, direction and control of the preparation of electoral rolls and
the conduct of all elections to the panchayats is vested in the State Election Commission.

The State Election Commissioner is appointed by the Governor, and his terms of service are also
determined by the Governor.

But, while he is appointed by the Governor, he can be removed only by the President, and not the
Governor.

Statement 3: It is not a subordinate body to the Election Commission of India. It has a separate
constitutional existence. It does not report annually to the ECI.

Q Source: Chapter 34: Panchayati Raj: Indian Polity: M Laxmikanth

11 With reference to pre-independent history of India, what was Komagata Maru?

A. It was an Indian steamship that sailed to Japan carrying Muslim passengers.


B. It was a Japanese steamship that sailed to Canada carrying immigrants from Punjab.
C. It was a Canadian steamship that sailed to Japan carrying Indian labourers.
D. It was an Indian steamship that sailed to Canada carrying people from Punjab.

Your Answer :
Correct Answer : B

Answer Justification :

Learning: The Komagata Maru incident provoked revolutionary nationalistic response in India.

It was a Japanese steamship that sailed from Hong Kong to Vancouver, Canada via Japan in
1914.

It was carrying 376 passengers who were immigrants from Punjab, India. Of these, only 24
were granted admittance in Canada when the ship docked in Vancouver.

At that time, Canada had laws restricting entry of migrants of Asian origin. Following a two
month stalemate, the ship and its 352 passengers were escorted out of the dock by the
Canadian military and forced to sail back to India.

At every port that the ship touched, it triggered off a wave of resentment and anger among
the Indian community and became the occasion for anti-British mobilization.

www.insightsias.com 8
Total Marks : 200
TEST - 23 (MOCK TEST - 18) Mark Scored : 0

Q Source: Chapter 12: India’s struggle for Independence: Bipin Chandra

12 The provisions of Part IX of the constitution relating to the Panchayats are not applicable to the

A. Fifth Schedule areas


B. The States of Nagaland, Meghalaya and Mizoram
C. States with International Borders
D. Both (a) and (b)

Your Answer :
Correct Answer : D

Answer Justification :

Learning: As per Article 243M of Part IX the constitution:

Part not to apply to certain areas.—(1) Nothing in this Part shall apply to the Scheduled Areas
referred to in clause (1) and the tribal areas referred to in clause (2), of article 244.

(2) Nothing in this Part shall apply to—

the States of Nagaland, Meghalaya and Mizoram;

the hill areas in the State of Manipur for which District Councils exist under any law for the
time being in force.

(3) Nothing in this Part—

relating to Panchayats at the district level shall apply to the hill areas of the District of
Darjeeling in the State of West Bengal for which Darjeeling Gorkha Hill Council exists under
any law for the time being in force;

Q Source:
http://www.mdoner.gov.in/sites/default/files/silo4_content/Constitutional%20Provision/243M.pdf

13 Consider the following statements. As per the constitution of India


1. The validity of any law relating to the delimitation of constituencies cannot be questioned in any
court.
2. No election to any panchayat is to be questioned except by an election petition presented to such
authority and in such manner as provided by the state legislature.

Which of the above is/are correct?


A. 1 only

www.insightsias.com 9
Total Marks : 200
TEST - 23 (MOCK TEST - 18) Mark Scored : 0

B. 2 only
C. Both 1 and 2
D. None

Your Answer :
Correct Answer : C

Answer Justification :

Justification: Statement 1: The 73rd Constitutional Amendment Act bars the interference by courts
in the electoral matters of panchayats.

It declares that the validity of any law relating to the delimitation of constituencies or the allotment
of seats to such constituencies cannot be questioned in any court.

Statement 2: This is to uphold the principle of fairness, impartiality and separation of legislature
and judiciary to an extent.

Q Source: Chapter 34: Panchayati Raj: Indian Polity: M Laxmikanth

14 Consider the following statements. Zero Budget Natural Farming


1. Does not involve the use of fertilizers and weedicides
2. Uses only those seeds that have been obtained from the same field

Which of the above is/are correct?


A. 1 only
B. 2 only
C. Both 1 and 2
D. None

Your Answer :
Correct Answer : D

Answer Justification :

Justification: As the name implies, it is a method of farming where the cost of growing and
harvesting plants is zero.

This means that farmers need not purchase fertilizers and pesticides in order to ensure the healthy
growth of crops.

It is, basically, a natural farming technique that uses biological pesticides instead of chemical-based
fertilizers.

Farmers use earthworms, cow dung, urine, plants, human excreta and such biological fertilizers for
crop protection. It reduces farmers’ investment. It also protects the soil from degradation.

Q Source:

www.insightsias.com 10
Total Marks : 200
TEST - 23 (MOCK TEST - 18) Mark Scored : 0

http://www.insightsonindia.com/2018/01/31/insights-daily-current-affairs-31-january-2018/

15 NASA launched the IMAGE satellite for

A. Imaging the Earth’s magnetosphere


B. Studying moons of most habitable regions in the solar system
C. Supplying essential interstellar connection to the International Space Station (ISS)
D. Investigating uninhabitable region on earth to understand the habitable conditions in other
celestial bodies

Your Answer :
Correct Answer : A

Answer Justification :

Background: NASA confirmed that it stumbled across radio transmissions from Imager for
Magnetopause-to-Aurora Global Exploration (IMAGE) science satellite, which lost contact with
mission control over 12 years ago.

Learning: IMAGE was the first satellite mission dedicated to imaging the Earth’s magnetosphere,
the region of space controlled by the Earth’s magnetic field and containing extremely tenuous
plasmas of both solar and terrestrial origin.

Invisible to standard astronomical observing techniques, these populations of ions and electrons
have traditionally been studied by means of localized measurements with charged particle
detectors, magnetometers, and electric field instruments.

Q Source: As mentioned above

16 Consider the following statements. As per the 73rd and 74th amendment to the Constitution of India
1. Every state is required to constitute a District Planning Committee (DPC).
2. The DPC consolidates the plans prepared by panchayats and municipalities in the district.
3. All the members of the DPC should be directly elected by the panchayat and municipalities in the
district.

Select the correct answer using the codes below.


A. 1 only
B. 1 and 2 only
C. 2 and 3 only
D. 1 and 3 only

Your Answer :
Correct Answer : B

Answer Justification :

Justification: The act lays down that only four-fifths of the members of a district planning

www.insightsias.com 11
Total Marks : 200
TEST - 23 (MOCK TEST - 18) Mark Scored : 0

committee should be elected by the elected members of the district panchayat and municipalities in
the district from amongst themselves.

The representation of these members in the committee should be in proportion to the ratio between
the rural and urban populations in the district.

The State Legislature makes provision for rest of the composition of DPC.

The chairperson of such committee shall forward the development plan to the state government.

Q Source: Chapter 35: Municipalities: Indian Polity: M Laxmikanth

17 Consider the following statements.


1. Green Initiatives for Future transport (GIFT) aims to develop and demonstrate hydrogen powered
IC engine and fuel cell based vehicles.
2. Green Initiative for Power Generation (GIP) envisages developing fusion reactors for supplying on-
grid power to the metropolitan areas.

Which of the above is/are correct?


A. 1 only
B. 2 only
C. Both 1 and 2
D. None

Your Answer :
Correct Answer : A

Answer Justification :

Justification: Hydrogen is suitable for meeting decentralised energy needs of the country’s
population. It can be used for telecom tower for providing back up power in rural areas, vehicular
transportation and Industrial applications

In this light, the National Hydrogen Energy Road Map (2006) is an industry driven planning process
that offers long-term energy solutions to the growing energy needs of India, while ensuring energy
security for the country.

The Road Map has identified two major initiatives; namely:

Green Initiatives for Future transport (GIFT) - It aims to develop and demonstrate hydrogen
powered IC engine and fuel cell based vehicles ranging from small two/three wheelers to heavy
vehicles through different phases of development.

Green Initiative for Power Generation (GIP) - It envisages developing and demonstrating hydrogen
powered IC engine/turbine and fuel cell based decentralized power generating systems ranging
from small watt capacity to MW size systems.

Q Source: Additional Research: SET A: Q36: CSP 2010

www.insightsias.com 12
Total Marks : 200
TEST - 23 (MOCK TEST - 18) Mark Scored : 0

18 Capital Adequacy Ratio (CAR) is decided by central banks and bank regulators to

A. Ensure that a greater part of the bank’s lending is allocated to long gestation period
infrastructure projects
B. Reduce the exposure of Indian banking to the Global economy
C. Prevent Commercial banks from taking excess leverage and becoming insolvent in the
process
D. All of the above

Your Answer :
Correct Answer : C

Answer Justification :

Justification: It is the ratio of a bank's capital in relation to its risk weighted assets and current
liabilities.

Option A: As credit creation (i.e. loan disbursals) of banks is highly risky business, the depositors’
money depends on the banks’ quality of lending. If the bank funding goes largely in risky projects
such as infrastructure, chances of bank insolvency is high.

Having adequate bank capital (compared to liabilities) helps to prevent bank failure, which arises in
case the bank cannot satisfy its obligations to pay the depositors and other creditors.

It is also a part of the overall Basel norms.

Since the whole payment system, public as well as private, depends on banks, safeguarding banks is
crucial to sound economic health.

Q Source: Revision of previous test: Chapter 12: Banking: Indian Economy: Ramesh Singh

19 Which of these functions CANNOT be devolved by the State Legislature to the Panchayats under the
73rd Constitutional Amendment Act?

A. Economic development
B. Implementation of land reforms
C. Social welfare
D. Subordinate Legislation for Community Living

Your Answer :
Correct Answer : D

Answer Justification :

Learning: Eleventh Schedule contains a list of functional items that can be placed within the
purview of panchayats by the State Legislature.

The State Legislature can devolve powers and responsibilities upon panchayats to prepare plans for

www.insightsias.com 13
Total Marks : 200
TEST - 23 (MOCK TEST - 18) Mark Scored : 0

economic development and social justice; and to perform some or all of the 29 functions listed in the
Eleventh Schedule of the Constitution.

Other Important items in the list are:

Minor irrigation and watershed development

Social welfare, including welfare of the handicapped and mentally retarded

Welfare of the weaker sections, and in particular, of the scheduled castes and the scheduled
tribes

Technical training and vocational education

Agriculture, including agricultural extension

Small-scale industries, including food processing industries

Roads, culverts, bridges, ferries, waterways and other means of communication

Rural electrification, including distribution of electricity

Education, including primary and secondary schools

Q Source: Chapter 34: Panchayati Raj: Indian Polity: M Laxmikanth

20 Which of the following forms a part of the capital account of India?


1. Remittances received from NRIs living in Saudi Arabia
2. A loan given to Myanmar
3. Foreign exchange earned from trade in Invisibles

Select the correct answer using the codes below.


A. 1 and 3 only
B. 2 only
C. 1, 2 and 3 only
D. 1 only

Your Answer :
Correct Answer : B

www.insightsias.com 14
Total Marks : 200
TEST - 23 (MOCK TEST - 18) Mark Scored : 0

Answer Justification :

Justification & Learning: Capital account shows the net change in asset ownership for a nation.

Statement 2: Issuing of external bonds, long-term investment abroad or by foreign residents (FDI),
or loans given to foreign countries FII etc form part of the capital account.

Statement 1 and 3: Trade of any sort, Interest payments and remittances form part of the current
account.

Since large capital inflows or outflows can have destabilizing effects on a nation's economy, many
countries have controls in place to regulate capital account flows.

Q Source: Chapter 15: External Sector of India: Indian Economy: Ramesh Singh

21 DDT, which is also known as an insecticide, has adverse affect on ecological health because

A. It is a persistent organic pollutant.


B. It increases susceptibility to fungal infection in birds.
C. It impairs metabolic functions in plants.
D. It depletes tropospheric ozone.

Your Answer :
Correct Answer : A

Answer Justification :

Justification: DDT is a persistent organic pollutant that is readily adsorbed to soils and sediments,
which can act both as sinks and as long-term sources of exposure contributing to terrestrial
organisms. So, (a) is correct.

Other pesticides impair metabolic functions in birds, rodents etc.

Learning: Depending on conditions, its soil half life can range from 22 days to 30 years. Routes of
loss and degradation include runoff, volatilization, photolysis and aerobic and anaerobic
biodegradation. Due to hydrophobic properties, in aquatic ecosystems DDT and its metabolites are
absorbed by aquatic organisms and adsorbed on suspended particles, leaving little DDT dissolved in
the water itself.

Q Source: Revision: Unit 14: 11th Chemistry NCERT

22 Which of the following is/are the objectives of the PESA Act, 1996?
1. To have village governance with participatory democracy and to make the gram sabha a nucleus of
all activities
2. To prevent panchayats at the higher level from assuming the powers and authority of panchayats at
the lower level of the gram sabha
3. To evolve a suitable administrative framework consistent with traditional practices

www.insightsias.com 15
Total Marks : 200
TEST - 23 (MOCK TEST - 18) Mark Scored : 0

Select the correct answer using the codes below.


A. 1 and 2 only
B. 2 only
C. 1 and 3 only
D. 1, 2 and 3

Your Answer :
Correct Answer : D

Answer Justification :

Justification: Some features of the Act are:

Legislation on Panchayats shall be in conformity with the customary law, social and religious
practices and traditional management practices of community resources;

Habitation or a group of habitations or a hamlet or a group of hamlets comprising a


community and managing its affairs in accordance with traditions and customs; and shall have
a separate Gram Sabha.

Every Gram Sabha to safeguard and preserve the traditions and customs of people, their
cultural identity, community resources and the customary mode of dispute resolution.

The Gram Sabhas have roles and responsibilities in approving all development works in the
village, identify beneficiaries, issue certificates of utilization of funds; powers to control
institutions and functionaries in all social sectors and local plans.

Q Source: Chapter 34: Panchayati Raj: Indian Polity: M Laxmikanth

23 With reference to the advantages and disadvantages of chlorine disinfection of water, consider the
following statements.
1. Chlorination of water is used to prevent the spread of waterborne diseases such as cholera based on
the strong oxidizing properties of Chlorine.
2. Chlorination can lead to formation of harmful trihalomethanes (THMs) in the water supply.

Which of the above is/are correct?


A. 1 only
B. 2 only
C. Both 1 and 2
D. None

Your Answer :
Correct Answer : C

www.insightsias.com 16
Total Marks : 200
TEST - 23 (MOCK TEST - 18) Mark Scored : 0

Answer Justification :

Justification: Statement 1: As a halogen, chlorine is a highly efficient disinfectant, and is added to


public water supplies to kill disease-causing pathogens, such as bacteria, viruses, and protozoans,
that commonly grow in water supply reservoirs, on the walls of water mains and in storage tanks.

Statement 2: Disinfection by chlorination can be problematic, in some circumstances. Chlorine can


react with naturally occurring organic compounds found in the water supply to produce compounds
known as disinfection by-products (DBPs). The most common DBPs are trihalomethanes (THMs) and
haloacetic acids (HAAs).

Trihalomethanes are the main disinfectant by-products created from chlorination with two
different types, bromoform and dibromochloromethane, which are mainly responsible for
health hazards.

Their effects depend strictly on the duration of their exposure to the chemicals and the
amount ingested into the body. In high doses, bromoform mainly slows down regular brain
activity, which is manifested by symptoms such as sleepiness or sedation.

Q Source: Additional Research: SET A: Q11: CSP 2010

24 Under ‘Import substitution’


1. Government protects the domestic industries from foreign competition
2. Exports are discouraged since they reduce domestic consumption and thus production.

Which of the above is/are correct?


A. 1 only
B. 2 only
C. Both 1 and 2
D. None

Your Answer :
Correct Answer : A

Answer Justification :

Justification: This policy is aimed at replacing or substituting imports with domestic production.

Statement 1: For e.g. in India, the government protected the domestic industries from foreign
competition, so that the same goods that are now imported can be produced domestically.

Protection from imports can take two forms: tariffs and quotas.

Statement 2: The best example would be ‘Make In India’ rather than ‘Import to India’. Firms that
used to export goods and services like mobile phones would manufacture in India itself after FDI.
This would reduce imports and build domestic manufacturing base.

www.insightsias.com 17
Total Marks : 200
TEST - 23 (MOCK TEST - 18) Mark Scored : 0

Export based industries are meant to generate an export surplus in the goods they export (e.g. Gem
and Jewelleries). If such industries are subsidized, they will not substitute imports from abroad (as
they don’t manufacture those goods), and would actually boost exports.

Q Source: Revision: Chapter on Industry: Indian Economy: Ramesh Singh

25 Consider the following statements.


1. Protecting a title or slogan requires the use of ‘patents’.
2. A trademark registration prevents others from producing the same good.

Which of the above is/are correct?


A. 1 only
B. 2 only
C. Both 1 and 2
D. None

Your Answer :
Correct Answer : D

Answer Justification :

Justification: S1: Protecting a title, slogan, or other short word phrase, generally requires
trademark, not Patent.

Both require registration, else trademark or patent cannot be claimed over the product (phrase,
logo etc.).

A patent for an invention is the grant of a property right to the inventor issued by the government.
It is the right to exclude others from making, using, offering for sale, or selling. Thus, it is more
restrictive than both copyrights and trademark.

S2: In trademark the same good can be made. However, it does not restrict others from innovating
in the patented subject-matter, as also in the case of copyrights (where the subject matter can be
written upon by others too).

Q Source: Additional Research: SET A: Q17: CSP 2010

26 A State Legislature may provide for the representation of which of the following persons in a
municipality?
1. MLAs or MPs eligible to get elected from the municipal area
2. Activists and scholars of the Environment
3. Teachers and graduates living within the municipal area
4. Persons having special knowledge or experience in municipal administration

Select the correct answer using the codes below.


A. 1 and 2 only
B. 2, 3 and 4 only

www.insightsias.com 18
Total Marks : 200
TEST - 23 (MOCK TEST - 18) Mark Scored : 0

C. 1 and 4 only
D. 1, 2 and 3 only

Your Answer :
Correct Answer : C

Answer Justification :

Learning: The provision is made to augment municipal administration and the municipality may
include:

Persons having special knowledge or experience in municipal administration without the right
to vote in the meetings of municipality.

The members of the Lok Sabha and the state legislative assembly representing constituencies
that comprise wholly or partly the municipal area.

The members of the Rajya Sabha and the state legislative council registered as electors within
the municipal area.

The chairpersons of committees (other than wards committees).

Q Source: Chapter 35: Municipalities: Indian Polity: M Laxmikanth

27 Consider the following statements.


1. The District court exercises jurisdiction both on original side and appellate side in civil and criminal
matters arising in the District.
2. The jurisdiction of district courts in both civil and criminal matters is usually set in concerned state
enactments.

Which of the above is/are correct?


A. 1 only
B. 2 only
C. Both 1 and 2
D. None

Your Answer :
Correct Answer : A

Answer Justification :

Justification: The territorial and pecuniary jurisdiction in civil matters is usually set in concerned
state enactments on the subject of civil courts.

www.insightsias.com 19
Total Marks : 200
TEST - 23 (MOCK TEST - 18) Mark Scored : 0

On the criminal side, jurisdiction is exclusively derived from the criminal procedure code. As per
this code the maximum sentence a Sessions Judge of district court may award to a convict is capital
punishment.

The district court has appellate jurisdiction over all subordinate courts situated in the district on
both civil and criminal matters.

Learning: Articles 233 to 237 in Part VI of the Constitution make provisions to regulate the
organization of subordinate courts and to ensure their independence from the executive, such as
appointment of District Judges, control over subordinate courts etc.

Q Source: Chapter 31: Subordinate Courts: Indian Polity: M Laxmikanth

28 Consider the following statements.


1. Phototrophic bio-film microbial fuel cell (MFCs) does not require the use of photosynthetic
microorganisms.
2. A MFC can generate energy using electrochemically active bacteria.

Which of the above is/are correct?


A. 1 only
B. 2 only
C. Both 1 and 2
D. None

Your Answer :
Correct Answer : B

Answer Justification :

Concept: A microbial fuel cell is a device that converts chemical energy to electrical energy by the
catalytic reaction of microorganisms.

A typical microbial fuel cell consists of anode and cathode compartments separated by a cation
(positively charged ion) specific membrane. In the anode compartment, fuel is oxidized by
microorganisms, generating CO2, electrons and protons.

Electrons are transferred to the cathode compartment through an external electric circuit, while
protons are transferred to the cathode compartment through the membrane.

Justification: Statement 1: Phototrophic biofilm MFCs (PBMFCs) are the ones that make use
photosynthetic microorganism like chlorophyta, cyanophyta etc., since they could carry out
photosynthesis and thus they act as both producers of organic metabolites and also as electron
donors

Statement 2: Some MFCs use electrochemically active bacteria to transfer electrons to the
electrode (electrons are carried directly from the bacterial respiratory enzyme to the electrode).

Some MFC's produce electric current by the bacterial decomposition of organic compounds in
water. But, there are microbial electrolysis cells (MEC) also which reverse this process to generate
www.insightsias.com 20
Total Marks : 200
TEST - 23 (MOCK TEST - 18) Mark Scored : 0

hydrogen or methane by applying a voltage to bacteria.

Q Source: Additional Research: CSP 2011

29 The island territories of the Andaman and Nicobar Islands provide India with a strategic presence at
the entrance to the

A. Isthmus of Oman
B. Strait of Malacca
C. Sea lines of Persia
D. Straits of Irrawaddy

Your Answer :
Correct Answer : B

Answer Justification :

Learning: India’s location at the base of continental Asia and at the top of the Indian Ocean
provides it a vantage point with respect to both, Central Asia and the Indian Ocean Region.

The island territories of the Andaman and Nicobar Islands, located 1,300 kms. away from the
mainland in the Bay of Bengal, provide India with a strategic presence at the entrance to the Strait
of Malacca, through which more than 60,000 shipping vessels transit each year.

Q Source: Chapter 9: India Yearbook 2016

www.insightsias.com 21
Total Marks : 200
TEST - 23 (MOCK TEST - 18) Mark Scored : 0

30 Consider the following statements with reference to Indo-Myanmar relations.


1. Myanmar has porous borders with our north eastern states.
2. India-Myanmar-Thailand Friendship Highway is functional now.
3. India and Myanmar are members of BIMSTEC and the Mekong-Ganga Cooperation.

Select the correct answer using the codes below.


A. 1 and 2 only
B. 2 and 3 only
C. 1 and 3 only
D. 1, 2 and 3

Your Answer :
Correct Answer : D

Answer Justification :

Learning: India’s engagement of Myanmar continues to be driven by shared economic and security
interest which include enhancing bilateral trade, strengthening democratic institutions in Myanmar
and eliminating threats from insurgent groups exploiting the porous borders with our north eastern
states.

A MOU on Border Cooperation, concluded between both countries in 2014, provides a strong
framework for security cooperation and intelligence exchange between India and Myanmar security
agencies in areas of coordinated patrolling, intelligence sharing, counter-insurgency, arms-
smuggling, drug, human and wildlife trafficking.

Statement 2: The route begins from Guwahati in India and connects to Mandalay in Myanmar, route
continues to Yangon in Myanmar and then to Mae Sot in Thailand, which then continues to
Bangkok.

Q Source:
https://economictimes.indiatimes.com/industry/indl-goods/svs/petrochem/india-increases-engageme
nt-with-myanmar/articleshow/62511213.cms

31 Consider the following about Nirbhay which is India’s first indigenously designed and developed long
range subsonic cruise missile.
1. It has the ability to pick out a target and attack it among multiple targets.
2. It is not capable of carrying nuclear warheads.
3. It can go round a target and perform several manoeuvres and then reengage it.

Select the correct answer using the codes below.


A. 1 and 3 only
B. 2 and 3 only
C. 1 and 2 only
D. 1, 2 and 3

Your Answer :
Correct Answer : A
www.insightsias.com 22
Total Marks : 200
TEST - 23 (MOCK TEST - 18) Mark Scored : 0

Answer Justification :

Justification: Nirbhay is an all-weather, low-cost, long-range cruise missile capable of carrying


conventional and nuclear war heads.

The two-stage missile Nirbhay is able to pick out a target and attack it among multiple targets. The
missile has a loitering capability, i.e., it can go round a target and perform several manoeuvres and
then re-engage it

It incorporates state-of-the-art inertial navigation system. Nirbhay was flight tested in 2014 and
2015.

Q Source: http://indianexpress.com/article/what-is/what-is-nirbhay-missile-4927514/

32 With reference to the Legislative Council of a state, consider the following statements.
1. Its strength depends on the strength of the assembly of the concerned state.
2. The Constitution fixes the maximum and the minimum strength of the Legislative Council.

Which of the above is/are correct?


A. 1 only
B. 2 only
C. Both 1 and 2
D. None

Your Answer :
Correct Answer : C

Answer Justification :

Justification: The maximum strength of the council is fixed at one-third of the total strength of the
assembly and the minimum strength is fixed at 40.

It means that the size of the council depends on the size of the assembly of the concerned state.
This is done to ensure the predominance of the directly elected House (assembly) in the legislative
affairs of the state.

Though the Constitution has fixed the maximum and the minimum limits, the actual strength of a
Council is fixed by Parliament.

Q Source: Chapter 29: State Legislature: Indian Polity: M Laxmikanth

33 The basic source of all minerals on earth is

A. Ocean basin churns


B. Solar winds and ion showers
C. Magma

www.insightsias.com 23
Total Marks : 200
TEST - 23 (MOCK TEST - 18) Mark Scored : 0

D. Asteroids and meteorites

Your Answer :
Correct Answer : C

Answer Justification :

Justification: Magma cools slowly as it rises towards Earth’s surface. It can take thousands to
millions of years to become solid when it is trapped inside Earth.

As the magma cools, solid rocks form (igneous).

These rocks are mixtures of minerals. Granite is a common rock that forms when magma cools.
Granite contains the minerals quartz, plagioclase feldspar, and potassium feldspar.

The same igneous rock can be eroded, weathered, transported, deposited and consolidated as
sedimentary rocks which are the source of non-metallic minerals.

Q Source: Revision: Fundamentals of Physical Geography: 11th NCERT

34 Who among the following officials of the state has a constitutional right to participate in any
proceedings of both the Houses of the State Legislature?

A. Governor of the State


B. Chief Justice, High Court
C. Advocate-General
D. State Election Commissioner

Your Answer :
Correct Answer : C

Answer Justification :

Learning: In addition to the members of a House, every minister and the advocate general of the
state have the right to speak and take part in the proceedings of either House or any of its
committees of which he is named a member, without being entitled to vote.

There are two other provisions related to this constitutional provision:

A minister can participate in the proceedings of a House, of which he is not a member.

A minister, who is not a member of either House, can participate in the proceedings of both
the Houses.

Q Source: Chapter 29: State Legislature: Indian Polity: M Laxmikanth

www.insightsias.com 24
Total Marks : 200
TEST - 23 (MOCK TEST - 18) Mark Scored : 0

35 Despite Sun being overhead throughout the year at the equator, it receives comparatively less
insolation than the tropics. This can be due to
1. Heavy cloudiness due to continuous evaporation that reduces the amount of solar insolation
2. Absence of Coriolis force at the equator

Which of the above is/are correct?


A. 1 only
B. 2 only
C. Both 1 and 2
D. None

Your Answer :
Correct Answer : A

Answer Justification :

Justification: Statement 1: Generally, at the same latitude the insolation is more over the continent
than over the oceans, because oceans are cloudier due to greater evaporation.

Similarly in equator, evaporation is high, cloudiness is high and rains are frequent. Clouds reflect
incoming solar insolation and send it back to space.

Statement 2: Zero coriolis force would mean that winds are not deflected with great force at the
equator, as they are the sub-tropics.

But, direction of wind will only affect the distribution of heat, and not the amount of insolation
actually received from the Sun.

Q Source: Revision: Fundamentals of Physical Geography: 11th NCERT

36 Water that rises to the surface as a result of Ocean Upwelling is typically

A. Colder and poor in nutrients


B. Warmer and poor in nutrients
C. Colder and rich in nutrients
D. Warmer and rich in nutrients

Your Answer :
Correct Answer : C

Answer Justification :

Learning: Winds blowing across the ocean surface push water away. Water then rises up from
beneath the surface to replace the water that was pushed away. This process is known as
“upwelling.”

Upwelling occurs in the open ocean and along coastlines. The reverse process, called

www.insightsias.com 25
Total Marks : 200
TEST - 23 (MOCK TEST - 18) Mark Scored : 0

“downwelling,” also occurs when wind causes surface water to build up along a coastline and
the surface water eventually sinks toward the bottom.

Water that rises to the surface as a result of upwelling is typically colder and is rich in
nutrients.

These nutrients “fertilize” surface waters, meaning that these surface waters often have high
biological productivity. Therefore, good fishing grounds typically are found where upwelling is
common.

Q Source: Additional Research: CSP 2011

37 Articles 371 to 371-J in Part XXI of the Constitution contain special provisions for

A. All states with Scheduled areas


B. North-eastern States
C. Least populated States
D. None of the above

Your Answer :
Correct Answer : D

Answer Justification :

Learning: These are 11 states – Gujarat, Sikkim, Goa, Karnataka, Arunachal Pradesh, Assam,
Nagaland, Manipur, Mizoram, Maharashtra and Andhra Pradesh (undivided).

The intention behind them is to meet the aspirations of the people of backward regions of the states
or to protect the cultural and economic interests of the tribal people of the states or to deal with the
disturbed law and order condition in some parts of the states or to protect the interests of the local
people of the states.

Originally, the constitution did not make any special provisions for these states. They have been
incorporated by the various subsequent amendments made in the context of reorganisation of the
states or conferment of statehood on the Union Territories.

Q Source: Chapter 33: Special Provisions for some states: Indian Polity: M Laxmikanth

38 Underground Coal Gasification (UCG) yields


1. Hydrogen
2. Methane
3. Carbon dioxide
4. Carbon monoxide

Select the correct answer using the codes below.

www.insightsias.com 26
Total Marks : 200
TEST - 23 (MOCK TEST - 18) Mark Scored : 0

A. 1 and 4 only
B. 2 and 3 only
C. 3 and 4 only
D. 1, 2, 3 and 4

Your Answer :
Correct Answer : D

Answer Justification :

Learning: Some time before, the Union Cabinet had approved a policy framework for development
of UCG in unexplored coal and lignite bearing areas in the country.

Also, Coal India Limited has planned to invest about Rs 15,000 crore for capital expenditure,
supercritical thermal power plant, solar energy and coal gasification this fiscal.

UCG is a clean coal extraction technology for extracting energy from the coal seams and lignite
resources which cannot be mined through traditional mining technology.

It is mainly an industrial process which converts coal into product gases such as methane,
hydrogen, CO and CO2 that are combusted for electricity generation.

The gasification process is carried out in non-mined coal seams using injection of oxidants which
brings the product gas to surface through production wells drilled from the surface.

Q Source:
http://www.livemint.com/Industry/41emZiHuUaNBCP8MIqFCQK/Coal-India-to-invest-Rs15000-crore
-this-fiscal-for-capex-o.html

39 As per the compulsory provisions of the 73rd amendment Act to the Constitution of India
1. All the members of panchayats at the village, intermediate and district levels shall be elected
directly by the people.
2. All expenditure within a Panchayat will be socially audited by the Panchayat Secretary.
3. Not less than one-third of the total number of seats will be reserved for women in PRIs.
4. All chairpersons in the Panchayati Raj Institutions (PRIs) will be nominated and appointed by the
Governor of the State.

Select the correct answer using the codes below.


A. 1, 2 and 3 only
B. 2, 3 and 4 only
C. 1 and 3 only
D. 1 and 4 only

Your Answer :
Correct Answer : C

Answer Justification :

www.insightsias.com 27
Total Marks : 200
TEST - 23 (MOCK TEST - 18) Mark Scored : 0

Learning: Statement 1 and 4: Only chairpersons of panchayats at the intermediate and district
levels shall be elected indirectly—by and from amongst the elected members thereof.

However, the chairperson of a panchayat at the village level shall be elected in such manner as the
state legislature determines.

Statement 2 and 3: Also, not less than one-third of the total number of offices of chairpersons in the
panchayats at each level shall be reserved for women.

Expenditure is socially audited when mandated by the law or government scheme, such as
MGNREGA. However, Gram Sabha is authorized to look into the financial records maintained by the
Gram Panchayat.

Q Source: Chapter 34: Panchayati Raj: Indian Polity: M Laxmikanth

40 Which of following characterizes the Western coasts of India?

A. Highly indented
B. Formation of wave-cut platforms in sea cliffs
C. Lagoons, deltas and swamps
D. Both (a) and (b)

Your Answer :
Correct Answer : D

Answer Justification :

Learning: The west coast of our country is a high rocky retreating coast. On the other hand,
erosional forms dominate in the west coast instead of depositional forms.

High rocks coast features:

Highly indented

Formation of wave-cut platforms in front of the sea cliff due to constant water erosional action

Material eroded in such manner, deposits along the off-shore forming wave terraces.

Several depositions like this results in the formation of barrier bars and spits that break block
the sea water forming a lagoon.

Low Sedimentary coast features:

Lagoons, deltas etc. found

www.insightsias.com 28
Total Marks : 200
TEST - 23 (MOCK TEST - 18) Mark Scored : 0

Lagoons eventually turn into swamps which turn into coastal plains

Storm and tsunami waves cause darastic changes in the supply of depositional material

Q Source: Revision: Chapter 7: Fundamentals of Physical Geography: 11th NCERT

41 Consider the following statements.


1. All the members of the Legislative Assembly of an Indian state are elected directly.
2. All except one-sixth of the members in the Legislative Council are elected directly by people.

Which of the above is/are correct?


A. 1 only
B. 2 only
C. Both 1 and 2
D. None

Your Answer :
Correct Answer : D

Answer Justification :

Justification: Statement 1: Some members of the legislative assemblies in Sikkim and Nagaland
are elected indirectly. Moreover, there is provision of the nomination of an Anglo-Indian in the
assembly as well. Under article 333, the Governor of an state is of the opinion that Anglo Indian
Community is not adequately represented in the state assembly, he / she can nominate the member.

Statement 2: Others are elected indirectly by either the state assembly or graduates/teachers etc.
We will cover this in detail in another question.

Q Source: Chapter 29: State Legislature: Indian Polity: M Laxmikanth

42 Abnormal accumulation of cold water occurring in the central and eastern Pacific Ocean and the
strengthening of Tropical pacific trade winds is an unusual occurrence associated with

A. El Nino
B. Indian Ocean Dipole
C. La Nina
D. Either (a) or (b)

Your Answer :
Correct Answer : C

Answer Justification :

Justification & Learning: La Niña refers to the extensive cooling of the central and eastern
tropical Pacific Ocean, often accompanied by warmer than normal sea surface temperatures (SSTs)

www.insightsias.com 29
Total Marks : 200
TEST - 23 (MOCK TEST - 18) Mark Scored : 0

in the western Pacific, and to the north of Australia.

Changes to the atmosphere and ocean circulation during La Niña events include:

Sustained cooler-than-usual SSTs across the central and eastern tropical Pacific Ocean.

Increased convection or cloudiness over tropical Australia, Papua New-Guinea, and Indonesia.

An increase in strength of the trade winds (easterlies) across the tropical Pacific Ocean (but
not necessarily in the Australian region).

Q Source: Revision: Glossary: Fundamentals of Physical Geography: 11th NCERT

43 Consider the following statements.


1. Arsenic contamination of groundwater is found in West Bengal.
2. High concentrations of fluoride in ground water are common in some of the semi-arid areas of
Rajasthan.
3. The sulphate content in groundwater is high in regions like Haridwar and Dehradun.

Select the correct answer using the codes below.


A. 1 and 2 only
B. 3 only
C. 1 and 3 only
D. 1, 2 and 3

Your Answer :
Correct Answer : D

Answer Justification :

Justification: Arsenic pollution of ground water in West Bengal was first reported in the early
eighties. The occurrence of arsenic is mainly due to two reasons: natural and anthropogenic.

High concentrations of fluoride in ground water are common in some of the semi-arid areas of
Rajasthan, southern Punjab, Gujarat, Karnataka, Tamil Nadu, Madhya Pradesh, and southern
Haryana. Groundwater in at least 387 districts has high nitrate levels.

Q Source: Revision: Unit 14: 11th Chemistry NCERT

44 Shigellosis is endemic throughout the world with the overwhelming majority of cases occurring in
developing countries and involving children less than five years of age. Which of the following
statements in this context is INCORRECT?

A. Shigellosis is an infectious disease caused by Virus.

www.insightsias.com 30
Total Marks : 200
TEST - 23 (MOCK TEST - 18) Mark Scored : 0

B. The symptoms of shigellosis are fever and watery diarrhea.


C. Shigella’s main mode of transmission is person-to-person contact.
D. The disease Shigellosis is usually resolved in a span of a week.

Your Answer :
Correct Answer : A

Answer Justification :

Justification: Shigella is bacteria. Shigellosis is held responsible for some 165 million cases of
severe dysentery with blood and mucus in the stools.

Because it has low infectious dose, the main mode of transmission is person-to-person contact.

It can also be transmitted through infected food and water.

Most who are infected with Shigella develop diarrhea, fever, and stomach cramps starting a
day or two after they are exposed to the bacteria. Shigellosis usually resolves in 5 to 7 days.

Some people who are infected may have no symptoms at all, but may still pass the Shigella
bacteria to others. The spread of Shigella can be stopped by frequent and careful
handwashing with soap and taking other hygiene measures.

Q Source: http://www.who.int/immunization/topics/shigella/en/

45 Disputes relating to the election of members of Parliament and state legislatures and the
Enforcement of fundamental rights of citizens comes under which of these jurisdictions of High
Court?

A. Writ Jurisdiction
B. Appellate Jurisdiction
C. Revisional Jurisdiction
D. Original Jurisdiction

Your Answer :
Correct Answer : D

Answer Justification :

Justification: Original Jurisdiction is the power of a high court to hear disputes in the first
instance, not by way of appeal.

It extends to the following:

www.insightsias.com 31
Total Marks : 200
TEST - 23 (MOCK TEST - 18) Mark Scored : 0

Matters of admirality, will, marriage, divorce, company laws and contempt of court.

Disputes relating to the election of members of Parliament and state legislatures.

Regarding revenue matter or an act ordered or done in revenue collection.

Enforcement of fundamental rights of citizens.

Cases ordered to be transferred from a subordinate court involving the interpretation of the
Constitution to its own file.

The four high courts (i.e., Kolkata, Bombay, Madras and Delhi High Courts) have original civil
jurisdiction in cases of higher value.

Q Source: Chapter 30: High Courts: Indian Polity: M Laxmikanth

46 The Government of India had decided to convert the famous headquarters of the Ghadr party into a
library and a museum. This famous place is known as

A. Gandhar Ashram
B. Mani Bhawan
C. Shaantigram Ashram
D. Yugantar Ashram

Your Answer :
Correct Answer : D

Answer Justification :

Learning: The Hindustan Association of the Pacific Coast, known as the Gadar Party was founded
in 1913 to free India from British slavery.

The headquarters of the association was established initially at 436 Hill Street, San Francisco and
named as “Yugantar Ashram.”

The GoI decided in 2013 to convert this memorial into a library and Museum.

http://www.sikhfoundation.org/people-events/gadar-memorial-in-san-francisco-to-be-museum-and-lib
rary/

The association began publishing a magazine, Gadar (revolt), for free distribution.

The Gadar publication exposed the British imperialism and called upon the Indian people to unite
and rise up against British rule and throw the British out of India. The publication Gadar, over a

www.insightsias.com 32
Total Marks : 200
TEST - 23 (MOCK TEST - 18) Mark Scored : 0

period of time, became well known among Indians and the Hindustan Association of the Pacific
Coast itself became known as the Gadar party.

Q Source: Chapter 12: Bipin Chandra: India’s Struggle for Independence

47 Consider the following about the Indian Home Rule movement.


1. It wanted complete independence from the British Empire and establish a constitutional democracy
in India.
2. Gandhiji boycotted the All India Home Rule League.
3. It worked as a joint movement of the Indian National Congress and the All India Muslim League.

Select the correct answer using the codes below.


A. 1 and 2 only
B. 2 and 3 only
C. 1 only
D. None of the above

Your Answer :
Correct Answer : D

Answer Justification :

Justification: Statement 1: Between 1916 and 1918, when the World War-I was gradually
approaching an end, prominent Indians like Muhammad Ali Jinnah, Bal Gangadhar Tilak, Sir S.
Subramania Iyer, Annie Besant decided to organise a national alliance of leagues across India.

It was specifically to demand Home Rule, or self-government within the British Empire for all of
India.

Statement 2: In 1920, the All India Home Rule League elected Mahatma Gandhi as its President. In
a year, the body would merge into the Indian National Congress to form a united Indian political
front.

Statement 3: It was run separately from both parties. Some members of both INC and AIML joined
the Home Rule movement.

Q Source: Chapter 13: Bipin Chandra: India’s Struggle for Independence

48 The appointment, posting and promotion of district judges in a state are made by the
1. Governor of the state
2. Chief Justice of State High Court
3. In consultation with the High Court
4. A collegium of senior-most judges of the High court
5. Chief Minister
6. In consultation with the Governor

Select the correct answer using the codes below.

www.insightsias.com 33
Total Marks : 200
TEST - 23 (MOCK TEST - 18) Mark Scored : 0

A. 1 and 3 only
B. 2 and 4 only
C. 5 and 6 only
D. 2 and 6 only

Your Answer :
Correct Answer : A

Answer Justification :

Justification: It is done by the governor of the state in consultation with the high court.

Appointment of persons (other than district judges) to the judicial service of a state is made by the
governor of the state after consultation with the State Public Service Commission and the high
court.

A person to be appointed as district judge should have the following qualifications:

He should not already be in the service of the Central or the state government.

He should have been an advocate or a pleader for seven years.

He should be recommended by the high court for appointment.

Q Source: Chapter 31: Subordinate Courts: Indian Polity: M Laxmikanth

49 Consider the following statements. The Indian National Congress (INC) was divided into two groups
1. Between extremists and moderates
2. In the year 1915 at the Lucknow session
3. Based on a resolution passed in the leadership of Pandit Nehru

Select the correct answer using the codes below.


A. 1 and 2 only
B. 1 only
C. 2 and 3 only
D. 1 and 3 only

Your Answer :
Correct Answer : B

Answer Justification :

Justification: This was in the year 1907 between extremists and moderates at the Surat Session of
the Congress.

www.insightsias.com 34
Total Marks : 200
TEST - 23 (MOCK TEST - 18) Mark Scored : 0

The period 1885-1905 was known as the period of the moderates as moderates dominated the INC.

These Moderates used petition, prayers, and protest to influence the (British) colonial government.
Moderates were not able to achieve their goals.

This created dissatisfaction among the people.

In 1907 the INC meeting was to be held in Pune and the extremists wanted Lala Lajpat Rai or Bal
Gangadhar Tilak as president. But moderates wanted Rash Behari Ghosh to be president.

This conflict, based on the differences in the larger anti-British ideology, lead to the split.

Q Source: Chapter 11: India’s Struggle for Independence: Bipan Chandra

50 Consider the following about the philosophy of Shri Aurobindo Ghosh.


1. He gave the concept of the Integral Yoga system.
2. He propounded the doctrine of no-Brahman or no-God.
3. He refuted the theory of evolution.

Select the correct answer using the codes below.


A. 1 only
B. 1 and 2 only
C. 1 and 3 only
D. 2 and 3 only

Your Answer :
Correct Answer : A

Answer Justification :

Justification: Statement 1: Integral Yoga is a system of Yoga that synthesizes six branches of
classical Yoga philosophy and practice: Hatha, Raja, Bhakti, Karma, Jnana, and Japa Yoga.

His main literary works are The Life Divine, which deals with theoretical aspects of Integral Yoga;
Synthesis of Yoga, which deals with practical guidance to Integral Yoga; and Savitri: A Legend and
a Symbol, an epic poem.

Statement 2: Sri Aurobindo argues that divine Brahman manifests as empirical reality through leela,
or divine play.

Instead of positing that the world we experience is an illusion (maya), Sri Aurobindo argues that
world can evolve and become a new world with new species, far above the human species just as
human species have evolved after the animal species.

Statement 3: Sri Aurobindo believed that Darwinism merely describes a phenomenon of the
evolution of matter into life, but does not explain the reason behind it, while he finds life to be
already present in matter, because all of existence is a manifestation of Brahman.

www.insightsias.com 35
Total Marks : 200
TEST - 23 (MOCK TEST - 18) Mark Scored : 0

He argues that nature (which he interpreted as divine) has evolved life out of matter and then
mind out of life.

All of existence, he argues, is attempting to manifest to the level of the supermind – that
evolution had a purpose.

He stated that he found the task of understanding the nature of reality arduous and difficult to
justify by immediate tangible results

Q Source: Chapter 12: India’s struggle for Independence: Bipin Chandra

51 Which of the following revolutionary organizations, of the colonial period, were based out of India?
1. Swadesh Sevak Home
2. Jashn-e-Hind
3. United India House

Select the correct answer using the codes below.


A. 1 and 2 only
B. 2 and 3 only
C. 1 and 3 only
D. 1, 2 and 3

Your Answer :
Correct Answer : C

Answer Justification :

Justification: Several nationalists were residing in countries abroad, but the discriminatory
policies of the host countries soon resulted in a flurry of political activity among Indian nationalists.

As early as 1907, Ramnath Purl, a political exile on the West Coast, issued a Circular-e-Azadi
(Circular of Liberty) in which he also pledged support to the Swadeshi Movement; Tarak Nath
Das in Vancouver started the Free Hindustan and adopted a very militant nationalist tone.

G.D. Kumar set up a Swadesh Sevak Home in Vancouver on the lines of the India House in
London and also began to bring out a Gurmukhi paper called Swadesh Sevak which
advocated social reform and also asked Indian troops to rise in revolt against the British.

In 1910, Tarak Nath Das and G.D. Kumar, by now forced out of Vancouver, set up the United
India House in Seattle in the US, where every Saturday they lectured to a group of twenty-
five Indian labourers.

www.insightsias.com 36
Total Marks : 200
TEST - 23 (MOCK TEST - 18) Mark Scored : 0

Close links also developed between the United India House group, consisting mainly of radical
nationalist students, and the Khalsa Diwan Society, and in 1913 they decided to send a
deputation to meet the Colonial Secretary in London and the Viceroy and other officials in
India.

Q Source: Chapter 12: India’s struggle for Independence: Bipin Chandra

52 Which of these offices/bodies is constitutionally authorized to allow a panchayat to levy tax?

A. President of India
B. Parliament
C. State legislature
D. Governor of the State

Your Answer :
Correct Answer : C

Answer Justification :

Justification: Following can be provided by the state legislature-

Authorise a panchayat to levy, collect and appropriate taxes, duties, tolls and fees;

Assign to a panchayat taxes, duties, tolls and fees levied and collected by the state
government;

Provide for making grants-in-aid to the panchayats from the consolidated fund of the state;
and

Provide for constitution of funds for crediting all moneys of the panchayats

Q Source: Chapter 34: Panchayati Raj: Indian Polity: M Laxmikanth

53 Consider the following about the Global Vaccine Action Plan (GVAP).
1. It is a framework to prevent millions of deaths by 2020 through more equitable access to existing
vaccines for people in all communities.
2. The GAVI Alliance is an important partner in the project.
3. It is based on public-private partnership.

Select the correct answer using the codes below.


A. 1 only
B. 1 and 2 only

www.insightsias.com 37
Total Marks : 200
TEST - 23 (MOCK TEST - 18) Mark Scored : 0

C. 2 and 3 only
D. 1, 2 and 3

Your Answer :
Correct Answer : D

Answer Justification :

Justification: Gavi, the Vaccine Alliance is a public-private partnership committed to saving


children's lives and protecting people's health by increasing access to immunisation in poor
countries.

GVAP was endorsed by the 194 Member States of the World Health Assembly in 2012 ― as a
framework for improved vaccination.

GVAP was the product of the DoV Collaboration (of GAVI), an unprecedented effort that brought
together development, health and immunization experts and stakeholders. The leadership of the Bill
& Melinda Gates Foundation, GAVI Alliance, UNICEF etc help sail the initiative.

Q Source: http://www.who.int/immunization/global_vaccine_action_plan/en/

54 Which of these major regions of the world are Coffee growing belts as well?
1. Latin America
2. Africa
3. Asia-Pacific

Select the correct answer using the codes below.


A. 1 and 2 only
B. 2 and 3 only
C. 3 only
D. 1, 2 and 3

Your Answer :
Correct Answer : D

Answer Justification :

Justification: The image contains required details.

www.insightsias.com 38
Total Marks : 200
TEST - 23 (MOCK TEST - 18) Mark Scored : 0

Q Source: Revision: 9th NCERT Geography

55 The first Ashram established by Gandhiji in India was

A. Kochrab Ashram, near Ahmedabad


B. Sabarmati Asharam
C. Khadi Pratishthan and Ashram at Sodepur, Kolkata
D. Bardoli Taluka Ashram

Your Answer :
Correct Answer : A

Answer Justification :

Learning: Kochrab was the first Ashram established by Gandhiji in India in 1915 CE, it was a
pioneering centre for students of Gandhian ideas to practise Satyagraha, self-sufficiency, Swadeshi,
work for the upliftment of the poor, women and untouchables, and public education and sanitation.

Sabarmati Ashram was the residence of Mahatma Gandhi for over twelve years (1917-1930 CE) and
laboratory for his social experiments, e.g. agriculture and literacy promotion.

It was also from here that in 1930, Mahatma Gandhi started his Dandi March.

This was not his first Ashram though. You can read about a lot of important Gandhian sites here.

Q Source: http://whc.unesco.org/en/tentativelists/5899/

56 INS Karanj is a

www.insightsias.com 39
Total Marks : 200
TEST - 23 (MOCK TEST - 18) Mark Scored : 0

A. Scorpene-class submarine
B. Designed to work in Polar Regions
C. Fitted with stealth features that makes it detection difficult
D. Both (a) and (c)

Your Answer :
Correct Answer : D

Answer Justification :

Background: The Scorpene submarines are a primary modernization requirement of the Indian
Navy, which is currently faced with an ageing submarine fleet.

The first one, INS Kalvari, a diesel-electric attack submarine was commissioned in 2014.

Justification: This is the third of the six Scorpene-class submarines being built by Mazagon Dock
Shipbuilders Limited (MDL) under the Project 75 programme of Indian navy.

Option B: It is designed to operate in all theatres, including the tropics. It is provided with all means
and communications to ensure interoperability with other components of a naval task.

It has superior stealth features such as advanced acoustic silencing techniques, low radiated noise
levels and hydrodynamically optimized shape. These stealth features give it an invulnerability,
unmatched by most submarine.

Q Source:
http://www.insightsonindia.com/2018/01/31/insights-daily-current-affairs-31-january-2018/

57 The “Provisions of the Panchayats (Extension to the Scheduled Areas) Act (PESA)”, 1996 is NOT
applicable to which of these states of India?

A. Rajasthan
B. Tamil Nadu
C. West Bengal
D. Odisha

Your Answer :
Correct Answer : C

Answer Justification :

Learning: Gram Sabhas have been constituted in every State as per the Panchayat Raj Act/PESA
Rules of the concerned State.

As of 2016, ten states have Fifth Schedule Areas. These are: Andhra Pradesh, Telangana,
Chhattisgarh, Gujarat, Himachal Pradesh, Jharkhand, Madhya Pradesh, Maharashtra, Odisha and
Rajasthan.

But, only Andhra Pradesh, Himachal Pradesh, Maharashtra and Rajasthan have framed their Rules
www.insightsias.com 40
Total Marks : 200
TEST - 23 (MOCK TEST - 18) Mark Scored : 0

for implementation of PESA.

You can read more about PESA here http://pib.nic.in/newsite/PrintRelease.aspx?relid=108320

Q Source: Chapter 34: Panchayati Raj: Indian Polity: M Laxmikanth

58 SkyRev360 is a project concerning

A. Global Satellite Navigation


B. Internet penetration
C. Airline industry
D. Paragliding promotion in hilly regions

Your Answer :
Correct Answer : C

Answer Justification :

Learning: It is a fully automated and comprehensive data gathering, invoicing and collection
service which eliminates inaccuracies, errors and delays for better revenue management.

SkyRev360 has been developed in collaboration with the International Air Transport Association
(IATA) under the Government of India’s Make in India initiative.

The solution gathers 100% of your flight data and produces comprehensive billing data.

SkyRev360 automates data processing, helping ANSPs overcome strenuous manual processes which
are prone to mistakes.

In this way:

The IATA team handles all queries and disputes related to billing and collection from your air
operators.

Incident reports, airspace usage, traffic volumes, economic values and more are always at
your fingertips.

Q Source:
http://www.insightsonindia.com/2018/01/31/insights-daily-current-affairs-31-january-2018/

59 With reference to the State polity, consider the following statements.


1. A person who is not a member of the state legislature can be appointed as Chief Minister and serve
a five-year term if the President agrees with the appointment.
2. A member of the State Legislative Council can be appointed as the Chief Minister if he fulfils all the
qualifications mentioned in the constitution of India.

www.insightsias.com 41
Total Marks : 200
TEST - 23 (MOCK TEST - 18) Mark Scored : 0

Which of the above is/are correct?


A. 1 only
B. 2 only
C. Both 1 and 2
D. None

Your Answer :
Correct Answer : B

Answer Justification :

Justification: Statement 1: He can serve if he is elected as an MLA/MLC within a period of six


months. This is similar to the case of the PM who has to become a MP within six months of
appointment if he wasn’t before.

Statement 2: He can be indirectly elected to the legislative council too and remain the Chief
Minister.

If he fails to get elected to the State legislature, he will cease to be the Chief Minister of the State.

For example, Bansilal and S B Chavan were appointed as Chief Ministers of Haryana and
Maharashtra respectively, even though they were not members of the state legislature.
Subsequently, they were elected to the state legislature.

Q Source: Chapter 27: Chief Minister: Indian Polity: M Laxmikanth

60 As per the recently unveiled China’s Arctic policy, China will


1. Participate in the development of Arctic shipping routes
2. Refrain from exploring or exploiting hydrocarbon resources in the Arctic for sustainable growth of
the region
3. Develop Arctic tourism

Select the correct answer using the codes below.


A. 1 and 2 only
B. 2 and 3 only
C. 1 and 3 only
D. 1 only

Your Answer :
Correct Answer : C

Answer Justification :

Justification: The policy will mainly focus its attention on four aspects:

www.insightsias.com 42
Total Marks : 200
TEST - 23 (MOCK TEST - 18) Mark Scored : 0

First, noting that “the Arctic shipping routes are likely to become important transport routes
for international trade” as a result of global warming, China plans to build a “Polar Silk Road”
by developing the Arctic shipping routes.

Second, China aims to participate “in the exploration for and exploitation of oil, gas, mineral
and other non-living resources” in the Arctic. However, the white paper also places a
particular emphasis on non-traditional energy sources.

Third, China will start to utilize fisheries and other living resources and participate in
conservation, since “the Arctic has the potential to become a new fishing ground in the
future.”

Fourth, China will develop Arctic tourism, which the paper described as “an emerging
industry.” China will support and encourage “its enterprises to cooperate with Arctic States in
developing tourism in the region” and conduct “training for and regulates Chinese tourism
agencies and professionals involved in Arctic tourism.”

Background: The land territories in the Arctic cover an area of about 8 million square kilometres,
whose sovereignty belongs to Canada, Denmark, Finland, Iceland, Norway, Russia, Sweden and the
US.

The Arctic Ocean has an area of over 12 million square kilometres, in which coastal and other
countries share maritime rights and interests according to international law.

Q Source: https://thediplomat.com/2018/01/china-issues-its-arctic-policy/

61 The Mission of NASA’s Global-Scale Observations of the Limb and Disk (GOLD) is to
1. Shed light on how the uppermost layers of Earth’s atmosphere can be affected by powerful space
and Earth-based weather events
2. Draw an infrared map of the innermost layers of the earth showing their structural and chemical
composition

Which of the above is/are correct?


A. 1 only
B. 2 only
C. Both 1 and 2
D. None

Your Answer :
Correct Answer : A

Answer Justification :

Background: Earth’s atmosphere is a complex, multi-layered protective shell that envelopes our
planet, and safeguards its inhabitants from dangerous space weather emanating largely from our

www.insightsias.com 43
Total Marks : 200
TEST - 23 (MOCK TEST - 18) Mark Scored : 0

Sun.

This tenuous region of the atmosphere has been known to undergo swift and significant
changes in less than an hour.

These fluctuations can be driven by the constant interactions between the ionosphere and
thermosphere, in conjunction with weather emanating from both Earth and space.

The complexity of these interactions makes it very difficult to predict when an atmospheric
change in the ionosphere and thermosphere will occur, and this can be a serious problem for
satellite communications.

Justification: GOLD is essentially an imaging spectrograph. Spectrographs are scientific


instruments that have been designed to break light down into its constituent wavelengths and to
measure their intensity.

By examining the data from such an instrument, scientists can determine a wide variety of
characteristics, including a target’s composition and temperature. GOLD will be tasked with
collecting far ultraviolet light data on Earth’s atmosphere.

GOLD will focus its attention on a relatively poorly-understood region of the upper atmosphere,
where the charged particles of the ionosphere mingle with the diffuse neutral gasses that make up
the thermosphere.

Q Source:
http://www.insightsonindia.com/2018/01/26/insights-daily-current-affairs-26-january-2018/

62 Consider the following statements. Almost all Indian coffee is


1. Shade grown
2. Grown as a monoculture
3. Alkaline in character

Select the correct answer using the codes below.


A. 1 only
B. 1 and 3 only
C. 1 and 2 only
D. 2 and 3 only

Your Answer :
Correct Answer : A

Answer Justification :

Justification: 70% of the coffee farms in India are very small, often less than 10 acres, and
frequently inter-cropped with pepper, cardamom, cinnamon, clove and nutmeg.

www.insightsias.com 44
Total Marks : 200
TEST - 23 (MOCK TEST - 18) Mark Scored : 0

The cup quality is generally low-acid, mild, and well balanced, with spicy notes and full body.

Learning: Currently, India is the sixth largest producer of coffee in the world, accounting for over
four percent of global coffee production.

Although India was one of the earliest growers of Arabica, it now produces a higher percentage of
Robusta.

And, India may soon become a net importer of Arabica coffee as the shift in coffee production in
India from Arabica to Robusta has been dramatic.

The rising cost of cultivation of Arabica due to lack of any scientific breakthrough in control of white
stem borer is a major problem.

Q Source: Additional Research: SET A: Q14: CSP 2010

63 Special safeguard mechanism (SSM) as under WTO relates to

A. Most Favoured Nation treatment dispute clause


B. WTO’s financial assistance to farmers that is not linked to prices or production
C. Licensing companies or individuals other than the patent owner to use the rights of the patent
D. Allowing developing countries to raise tariffs temporarily to deal with import surges or price
falls

Your Answer :
Correct Answer : D

Answer Justification :

Learning: WTO’s Special Safeguard Mechanism (SSM) is a protection measure allowed for
developing countries to take contingency restrictions against agricultural imports that are causing
injuries to domestic farmers.

The contingency measure is imposition of tariff if the import surge causes welfare loss to the
domestic poor farmers. The design and use of the SSM is an area of conflict under the WTO.

At the Doha Ministerial Conference, the developing countries were given a concession to adopt a
Special Safeguard Mechanism (SSM) besides the existing safeguards (like the Special Agricultural
Safeguard or the SSG).

This SSM constituted an important part of the promises offered to the developing world at Doha
(known as Doha Development Agenda) and the Doha MC became known as a development round.

The SSG was available to all countries- both developing and developed whereas the SSM is
allowable only to the developing countries.

Q Source: Additional Research: SET A: Q15: CSP 2010

www.insightsias.com 45
Total Marks : 200
TEST - 23 (MOCK TEST - 18) Mark Scored : 0

64 ‘Deflation’ in an economy would necessarily suggest decline in the


1. Prices of all the goods
2. GDP of the country

Which of the above is/are correct?


A. 1 only
B. 2 only
C. Both 1 and 2
D. None

Your Answer :
Correct Answer : D

Answer Justification :

Justification: Statement 1: Deflation shows a general decline in prices of a selected basket of


goods. It is not necessary that prices of all goods must have declined.

Statement 2: In economics, deflation is a decrease in the general price level of goods and services.

Deflation occurs when the inflation rate falls below 0% (a negative inflation rate). This should not be
confused with disinflation, a slow-down in the inflation rate (i.e., when inflation declines to lower
levels).

So, in a situation of deflation GDP may be growing, reducing or constant.

Q Source: Additional Research: SET A: Q19: CSP 2010

65 The "commanding heights" of the economy, as known in post-independent India goes along with
which of these economic philosophies?

A. Capitalist
B. Socialist
C. Communist
D. Anarchic

Your Answer :
Correct Answer : B

Answer Justification :

Justification: The Economic Model of India assigned a pivotal role to its public sector, which came
to control "commanding heights" of the economy after the Indian parliament adopted a resolution in
1956 to achieve a "socialistic pattern of society" as a national goal.

In pursuance to this goal, a large number of public sector units (PSUs) were set up in different
sectors of the economy.

www.insightsias.com 46
Total Marks : 200
TEST - 23 (MOCK TEST - 18) Mark Scored : 0

The expansion of the public sector did not take place only in a traditional direction i.e. (correct
provision of public goods and infrastructure), but also to non-traditional command over the
economy, in particular: (a) Nationalization to occupy the "commanding heights" of the economy and
regulations to protect them from internal and foreign competition, (b) Widespread subsidies,
explicit and implicit, and (c) Administered prices and related regulations.

Q Source: Revision: Chapter on Industry: Indian Economy: Ramesh Singh

66 Consider the following statements. A cantonment board is


1. Created by the State government
2. Established for municipal administration of defence personnel residing in civilian areas
3. Works under the administrative control of the defence ministry

Select the correct answer using the codes below.


A. 1 and 2 only
B. 2 and 3 only
C. 3 only
D. 1 and 3 only

Your Answer :
Correct Answer : B

Answer Justification :

Justification: A cantonment board is established for municipal administration for civilian


population in the cantonment area.

It is set up under the provisions of the Cantonments Act of 2006—a legislation enacted by the
Central government.

Statement 2: It works under the administrative control of the defence ministry of the Central
government.

Thus, unlike some other types of urban local bodies, which are created and administered by the
state government, a cantonment board is created as well as administered by the Central
government.

Learning: The executive officer implements all the resolutions and decisions of the board and its
committees. He belongs to the central cadre established for the purpose.

Q Source: Chapter 35: Municipalities: Indian Polity: M Laxmikanth

67 It is one of the largest national parks of India and the Great Indian bustard is commonly found in the
region. Despite a fragile ecosystem there is an abundance of birdlife here. It is

A. Desert National Park


B. Hemis National Park

www.insightsias.com 47
Total Marks : 200
TEST - 23 (MOCK TEST - 18) Mark Scored : 0

C. Neora Valley National Park


D. Khangchendoga National park

Your Answer :
Correct Answer : A

Answer Justification :

Justification: The major landform of the Desert National Park consists of craggy rocks and
compact salt lake bottoms, intermedial areas and fixed dunes.

It has a collection of fossils of animals and plants of 180 million years old. Some fossils of dinosaurs
of 6 million years old have been found in the area

The region is a haven for migratory and resident birds of the desert. The endangered Indian bustard
is the major attraction of Desert National Park.

Gadsisar Sagar Tank tank is among the tourist places in Jaisalmer, Rajasthan. Thousands of
migratory birds come to this place every year.

Hemis is the largest national park of India.

Q Source: Additional Research: SET A: Q64: CSP 2010

68 Consider the following statements. El Nino Modoki


1. is a Tropospheric phenomenon congruent with the occurrence of El- Nino on the hydrosphere
2. Affects the land temperature rather than the sea temperature as in the case with El Nino

Which of the above is/are correct?


A. 1 only
B. 2 only
C. Both 1 and 2
D. None

Your Answer :
Correct Answer : D

Answer Justification :

Learning: El Nino Modoki is a coupled ocean-atmosphere phenomenon in the tropical Pacific. It is


different from another coupled phenomenon in the tropical Pacific namely, El Nino. Conventional El
Nino is characterized by strong anomalous warming in the eastern equatorial Pacific.

However, El Nino Modoki is associated with strong anomalous warming in the central tropical
Pacific and cooling in the eastern and western tropical Pacific. Associated with this distinct
warming and cooling patterns the teleconnections are very different from teleconnection patterns of
the conventional El Nino.

The El Niño Modoki significantly influences regional climates in China, Japan and US.
www.insightsias.com 48
Total Marks : 200
TEST - 23 (MOCK TEST - 18) Mark Scored : 0

Q Source: Additional Research: SET A: Q4: CSP 2010

69 The metal central to this age finds frequent mention in the Vedas and the age itself follows after the
Chalcolithic age. It can be

A. Palaeolithic Age
B. Iron Age
C. Copper-stone
D. Later Stone Age

Your Answer :
Correct Answer : B

Answer Justification :

Learning: The Chalcolithic age is followed by Iron Age. Iron is frequently referred to in the Vedas.
The Iron Age of the southern peninsula is often related to Megalithic Burials. Megalith means Large
Stone.

The burial pits were covered with these stones. Such graves are extensively found in South India.

The early period of the age is characterized by the widespread use of iron or steel. The adoption of
these materials coincided with other changes in society, including differing agricultural practices,
religious beliefs and artistic styles.

Q Source: Revision: Chapter 2: 11th Std. TamilNadu History Textbook

70 Consider the following statements.


1. The Returning Officer of a parliamentary or assembly constituency is responsible for the conduct of
elections in the parliamentary or assembly constituency.
2. The Election Commission of India nominates or designates an officer of the Government or a local
authority as the Returning Officer.
3. The Returning Officer is responsible for the preparation of electoral rolls for a parliamentary or
assembly constituency.

Select the correct answer using the codes below.


A. 1 and 2 only
B. 2 and 3 only
C. 2 only
D. 1 and 3 only

Your Answer :
Correct Answer : A

Answer Justification :

Justification: S1 and 2: No more information is required.

www.insightsias.com 49
Total Marks : 200
TEST - 23 (MOCK TEST - 18) Mark Scored : 0

Statement 3: The Electoral Registration officer is responsible for the preparation of electoral rolls
for a parliamentary / assembly constituency.

The Presiding Officer with the assistance of polling officers conducts the poll at a polling station.

The Election Commission of India, in consultation with the State / UT Government, appoints an
Officer of the Government or the Local Authorities as the Electoral Registration Officer.

It also nominates officers of Government as Observers (General Observers and Election Expenditure
Observers)

Q Source: http://eci.nic.in/eci_main1/election-machinery.aspx

71 Abhidhamma Pitakas contain

A. Contemporary research into the fundamental Buddhist ideas available in different languages
B. Teachings attributed to the Buddha or his close companions
C. Philosophical and psychological discourse and interpretation of Buddhist doctrine
D. Rules and regulations of monastic life ranging from dress code and dietary rules to
prohibitions of certain personal conducts

Your Answer :
Correct Answer : C

Answer Justification :

Justification: The Tripitakas of Buddhism are known as the Sutta, the Vinaya and the Abhidhamma
Pitakas.

Out of them, the Abhidhamma Pitaka is a detailed scholastic reworking (interpretation and
comment) of material appearing in the Suttas, according to schematic classifications. It does
not contain systematic philosophical treatises, but summaries or enumerated lists.

Abhidhamma has been variously described as philosophy, psychology, and metaphysics.

Tradition holds that the Buddha thought out the Abhidhamma immediately after his
enlightenment then taught it to the gods some years later. Later the Buddha repeated it to
Sariputta who then handed it on to his disciples.

Scholars, however, generally date the Abhidhamma works to originating sometime around the
third century BCE, 100 to 200 years after the death of the Buddha. Therefore, the seven
Abhidhamma works are generally claimed by scholars not to represent the words of the
Buddha himself, but those of disciples and scholars.

www.insightsias.com 50
Total Marks : 200
TEST - 23 (MOCK TEST - 18) Mark Scored : 0

Q Source: Revision: Chapter 4: 11th Std. TamilNadu History Textbook

72 Which of the following cities is known as India's healthcare capital?

A. Bangalore
B. Bhopal
C. Allepey
D. Chennai

Your Answer :
Correct Answer : D

Answer Justification :

Learning: Chennai attracts about 45 percent of health tourists from abroad arriving in the country
and 30 to 40 percent of domestic health tourists in India.

Dental clinics have also attracted dental care tourism to Chennai. The statistics also show that TN
attracted the third highest number of domestic tourists in 2009.

Factors behind the tourists inflow in the city include low costs, little to no waiting period, and
facilities offered at the specialty hospitals in the city.

Of all the hospital beds in the city, only half is used by the city's population with the rest being
shared by patients from other states of the country and foreigners.

Q Source: Revision: 10th NCERT: Geography

73 The subject of ‘urban local government’ in Union Territories (UT) is dealt by the
#00000

A. Union Ministry of Minority Affairs


B. Union Ministry of Home Affairs
C. Union Ministry of Defence
D. Union Ministry of Urban Development

Your Answer :
Correct Answer : B

Answer Justification :

Justification: Under the Government of India (Allocation of Business) Rules, 1961, certain subjects
pertaining to the UTs have been allocated to the Ministry of Home Affairs.

The Union Territory Division of the MoHA deals with all legislative and constitutional matters
relating to Union Territories, including National Capital Territory of Delhi.

Option C: Ministry of Defence deals with cantonments boards only in the context of urban local

www.insightsias.com 51
Total Marks : 200
TEST - 23 (MOCK TEST - 18) Mark Scored : 0

government.

Option D: Ministry of Urban Development deals with the same subject in States of India.

Q Source: Chapter 35: Municipalities: Indian Polity: M Laxmikanth

74 Oil extraction in Arctic may lead to release of


1. Black carbon
2. Methane
3. Carbon Monoxide

Select the correct answer using the codes below.


A. 2 and 3 only
B. 2 only
C. 1 and 2 only
D. 1 only

Your Answer :
Correct Answer : C

Answer Justification :

Justification: This is a lateral explanation.

Oil extraction in Arctic may release black carbon and methane leading to global warming and
thawing of Arctic ice. It subsequently causes climate change. Some studies estimate the cost
involved as $60 trillion to the world economy.

Apart from the usual emission issues from the Arctic Permafrost (a concept covered in earlier tests),
many of the boundary lines in the Arctic remain to be fully demarcated as under EEZ. Therefore,
disputes can arise on resource extraction in the Arctic.

The short growing season and the low diversity of living organisms make the Arctic ecosystem
vulnerable to global warming, climate change, exploratory activities etc.

Q Source: https://www.worldwildlife.org/threats/oil-and-gas-development

75 Consider the following statements.


1. No lawyer or mediator or councillor is required for arbitration.
2. Decision reached through arbitration can be binding and need not be approved by the courts.

Which of the above is/are correct?


A. 1 only
B. 2 only
C. Both 1 and 2
D. None

www.insightsias.com 52
Total Marks : 200
TEST - 23 (MOCK TEST - 18) Mark Scored : 0

Your Answer :
Correct Answer : B

Answer Justification :

Justification: Arbitration is a procedure in which a dispute is submitted, by agreement of the


parties, to one or more arbitrators who make a binding decision on the dispute. In choosing
arbitration, the parties opt for a private dispute resolution procedure instead of going to court.

Alternative dispute resolution in India is not a new concept and it was in existence even under the
previous Arbitration Act, 1940.

The Arbitration and Conciliation Act, 1996 was enacted and then the ordinance in 2015.

Decision reached through arbitration can be binding and need not be approved by the courts.
However, the decisions can be challenged in some cases in the courts.

Q Source:
https://economictimes.indiatimes.com/industry/healthcare/biotech/pharmaceuticals/delhi-high-court-
set-to-deliver-verdict-on-daiichi-sankyos-rs-3500-crore-arbitration-award/articleshow/62712806.cms

76 In post-independent India, the Mahalanobis model of economic growth adopted laid stress on

A. Focus on consumer goods instead of capital goods


B. Development of agriculture
C. Reliance on heavy industries and import substitution
D. Promoting Foreign Direct Investment

Your Answer :
Correct Answer : C

Answer Justification :

Learning: At the time of formulation of the 2nd five year plan, Mahalanobis showed that to achieve a
rapid long- term rate of growth it would be essential to devote a major part of the investment outlay
to building of basic heavy industries.

The pillars of Nehru - Mahalanobis strategy were - (a) high savings rate, (b) heavy industry bias, (c)
protectionist policies and public sector, (d) import substitution, and (e) socialistic pattern of society.

However, in critique of the Mahalanobis heavy industry development strategy, Professors Vakil and
Brahmanand of Bombay University put forward a wage-goods (consumer goods) model of
development and suggested a development strategy which accorded a top priority to agriculture
and other wage-goods industries in sharp contrast to the Mahalanobis heavy industry biased
strategy of development.

Q Source: Additional Research: Chapter on Industry: Indian Economy: Ramesh Singh

www.insightsias.com 53
Total Marks : 200
TEST - 23 (MOCK TEST - 18) Mark Scored : 0

77 A microwave oven can best heat

A. High fibre foods


B. Dry fruits
C. High water content food item
D. Nutrition rich foods

Your Answer :
Correct Answer : C

Answer Justification :

Justification: A microwave oven heats food by passing microwave radiation through it. Microwaves
are a form of non-ionizing electromagnetic radiation with a frequency higher than ordinary radio
waves but lower than infrared light. Microwave ovens use frequencies in one of the ISM (industrial,
scientific, medical) bands, which are reserved for this use, so they do not interfere with other vital
radio services.

It induces polar molecules in the food to rotate and produce thermal energy. Since water is a polar
molecule, the most appropriate option here is (c).

They are also useful for rapid heating of otherwise slowly prepared cooking items, such as hot
butter, fats, and chocolate. Unlike conventional ovens, microwave ovens usually do not directly
brown or caramelize food, since they rarely attain the necessary temperatures.

Q Source: Additional Research: SET A: Q2: CSP 2010

78 Consider the following statements.


1. The form of government during the Sangam period in South India was a Federal republic.
2. Each of the Sangam dynasties had a royal Emblem.
3. The head of the Government in the Sangam age was assisted by a Council of Ministers.

Select the correct answer using the codes below.


A. 1 and 2 only
B. 2 and 3 only
C. 1 and 3 only
D. 1, 2 and 3

Your Answer :
Correct Answer : B

Answer Justification :

Justification: Statement 1: Hereditary monarchy was the form of government during the Sangam
period. The king also used to take the advice of his minister, court-poet and the imperial court or
avai.

S2: Each of the Sangam dynasties had a royal Emblem - carp for the Pandyas, tiger for the Cholas

www.insightsias.com 54
Total Marks : 200
TEST - 23 (MOCK TEST - 18) Mark Scored : 0

and bow for the Cheras.

The king was assisted by a large body of officials who were divided into five councils. They were
ministers (amaichar), priests (anthanar), military commanders (senapathi), envoys (thuthar) and
spies (orrar). The military administration was also efficiently organized during the Sangam Age.

Q Source: Page 89: Chapter 8: 11th Std. TamilNadu History Textbook

79 Consider the following statements. Tax policy of a government directly affects the
1. Level of savings and investment in the economy
2. Attractiveness of foreign investment

Which of the above is/are correct?


A. 1 only
B. 2 only
C. Both 1 and 2
D. None

Your Answer :
Correct Answer : C

Answer Justification :

Justification: Taxes directly affect the savings of individuals because high taxes erode income, and
low taxes help the individual have more savings.

If firms pay less taxes, they save more and invest more which affects the aggregate level of
investment in the economy.

As investment affects the output (GDP), taxes also have an influence over the per capita income and
thus the attractiveness of the foreign investors who look for good returns in the economy.

Taxes also affect the prices of goods and services as factor cost (production cost) is affected thereby
affecting incentives and behaviour of the economic activities, etc.

Q Source: Revision: Indian Economy: Ramesh Singh

80 Most of the Gupta kings encouraged


1. Brahmanism
2. Religious education of women
3. Equal treatment of all varnas

Select the correct answer using the codes below.


A. 1 only
B. 2 and 3 only
C. 1 and 3 only
D. 1 and 2 only

www.insightsias.com 55
Total Marks : 200
TEST - 23 (MOCK TEST - 18) Mark Scored : 0

Your Answer :
Correct Answer : A

Answer Justification :

Justification & Learning: Statement 1: The Brahmins occupied the top ladder of the society.

It had two branches - Vaishnavism and Saivism.

Most of the Gupta kings were Vaishnavaites. They performed Aswamedha sacrifices.

Statement 2 and 3: The practice of untouchability had slowly begun during this period.

The position of women had also become miserable during the Gupta period. They were prohibited
from studying the religious texts like the Puranas.

The subjection of women to men was thoroughly regularized. But it was insisted that they should be
protected and generously treated by men.

Q Source: Revision: Chapter 9: 11th Std. TamilNadu History Textbook

81 Which of the following bacterium may be used for bio-remediation operations?


1. Haemophilus
2. Mycobacterium
3. Rhodococcus
4. Ralstonia

Select the correct answer using the codes below.


A. 1 and 2 only
B. 2, 3 and 4 only
C. 1 and 3 only
D. 1, 2, 3 and 4

Your Answer :
Correct Answer : D

Answer Justification :

Learning: Bacteria are widely diverse organisms, and thus make excellent players in
biodegradation and bioremediation. There are few universal toxins to bacteria, so there is likely an
organism able to break down any given substrate, when provided with the right conditions
(anaerobic versus aerobic environment, sufficient electron donors or acceptors, etc.).

Fast development of molecular microbiological tool has enabled the identification of many un-
culturable microbes and therefore extended the list of microbial species with petroleum
hydrocarbon degrading abilities.

Species of Pseudomonas, Mycobacterium, Haemophilus, Rhodococcus, Paenibacillus and Ralstonia,


are some of the most extensively studied bacteria for their bioremediation capability.

www.insightsias.com 56
Total Marks : 200
TEST - 23 (MOCK TEST - 18) Mark Scored : 0

Q Source: Additional Research: SET A: Q57: CSP 2010

82 If any question arises whether a matter, concerning the state, falls within the Governor’s discretion
or not, whose decision shall be final?

A. President of India
B. Governor of the State
C. State Legislature
D. Chief Minister

Your Answer :
Correct Answer : B

Answer Justification :

Justification: The Council of Ministers aids and advises the Governor in the exercise of his
functions, except in so far as he is required to act in his discretion.

If any question arises whether a matter falls within the Governor’s discretion or not, decision of the
Governor shall be final, and the validity of anything done by the Governor shall not be called in
question on the ground that he ought or ought not to have acted in his discretion.

Also, the constitution says that the advice tendered by Ministers to the Governor shall not be
inquired into in any court.

This provision emphasises the intimate and the confidential relationship between the governor and
the ministers.

Q Source: Chapter 28: State Council of Ministers: Indian Polity: M Laxmikanth

83 Consider the following statements. In a closed economy


Assertion (A): Balance of Payments is zero.
Reason (R): No external transactions from outside the nation’s economy take place.

In the context of the above, which of these is correct?


A. A is correct, and R is an appropriate explanation of A.
B. A is correct, but R is not an appropriate explanation of A.
C. A is correct, but R is incorrect.
D. Both A and R are incorrect.

Your Answer :
Correct Answer : A

Answer Justification :

Justification: A closed economy is self-sufficient, meaning that no imports are brought in and no
exports are sent out. The goal is to provide consumers with everything that they need from within
the economy's borders. A closed economy is the opposite of an open economy, in which a country
www.insightsias.com 57
Total Marks : 200
TEST - 23 (MOCK TEST - 18) Mark Scored : 0

will conduct trade with outside regions.

So, if no capital or goods/services are imported, exported, the BoP will be zero.

Q Source: Additional Research: CSP 2011

84 If the Legislative council of a State rejects the bill passed once by the Legislative Assembly
1. The bill is passed again by the assembly and transmitted to the Council only after the Governor’s
consent
2. The bill lapses and does not become an act
3. A joint sitting of both houses is immediately to be called by the Governor
4. The Governor reserves the bill for Presidential review

Select the correct answer using the codes below.


A. 1 or 2 or 3
B. 3 or 4
C. 2 only
D. None of the above

Your Answer :
Correct Answer : D

Answer Justification :

Justification: If the council passes the bill without amendments or the assembly accepts the
amendments suggested by the council, the bill is deemed to have been passed by both the Houses
and the same is sent to the the governor for his assent.

On the other hand, if the assembly rejects the amendments suggested by the council or the
council rejects the bill altogether or the council does not take any action for three months,
then the assembly may pass the bill again and transmit the same to the council.

If the council rejects the bill again or passes the bill with amendments not acceptable to the
assembly or does not pass the bill within one month, then the bill is deemed to have been
passed by both the Houses in the form in which it was passed by the assembly for the second
time.

Therefore, the ultimate power of passing an ordinary bill is vested in the assembly. At the
most, the council can detain or delay the bill for a period of four months.

Q Source: Chapter 29: State Legislature: Indian Polity: M Laxmikanth

85 Consider the following statements.


1. Alkaline soils are difficult to take into agricultural production due to their high infiltration rate that

www.insightsias.com 58
Total Marks : 200
TEST - 23 (MOCK TEST - 18) Mark Scored : 0

does not allow water to stand.


2. Waterlogging of the soil prevents air from getting inside resulting in a decline in the productivity of
the land.

Which of the above is/are correct?


A. 1 only
B. 2 only
C. Both 1 and 2
D. None

Your Answer :
Correct Answer : B

Answer Justification :

Justification: Statement 1: Alkaline soils are difficult to take into agricultural production. Due to
the low infiltration capacity, rain water stagnates on the soil easily and, in dry periods, cultivation is
hardly possible without copious irrigated water and good drainage.

Statement 2: Soil may be regarded as waterlogged when it is nearly saturated with water much of
the time such that its air phase is restricted and anaerobic conditions prevail. Various crops need
air (specifically, oxygen) to a greater or lesser depth in the soil. Waterlogging of the soil stops air
getting in and reduces the produce.

Q Source: Additional Research: CSP 2011

86 The Principle of “Right Faith” as propounded by Mahavira Jaina is that

A. That all sentient beings are destined to salvation


B. All must understand their role in spiritual life and strictly follow the doctrine of ahimsa
C. Unshaken belief in the teachings and wisdom of Mahavira
D. Not believing in the theory of a God or a creator

Your Answer :
Correct Answer : C

Answer Justification :

Justification: The three principles or Triratnas of Jainism are right faith, right knowledge and right
conduct.

Triratnas—3 Gems of Jainism

• Samyak Shradha/ Vishwas (Right Faith) — Belief of Tirthankaras

• Samyak man or Gyan (Right Knowledge) — Knowledge of Jam belief

• Samyak Karma/Acharana (Right Action/Conduct) — Practise of 5 vows of Jainism

www.insightsias.com 59
Total Marks : 200
TEST - 23 (MOCK TEST - 18) Mark Scored : 0

Learning: Mahavira regarded all objects, both animate and inanimate, have souls and various
degrees of consciousness. They possess life and feel pain when they are injured.

Even the practice of agriculture was considered sinful as it causes injury to the earth, worms and
animals. Similarly the doctrine of asceticism and renunciation was also carried to extreme lengths
by the practice of starvation, nudity and other forms of self-torture.

Q Source: Revision: 11th Std. TamilNadu History Textbook

87 A Money bill can be introduced in the Legislative Council

A. Only with the prior recommendation of the Governor


B. Only with the Chairman’s prior permission
C. Only by a Minister
D. None of the above is correct.

Your Answer :
Correct Answer : D

Answer Justification :

Justification and Learning: A Money Bill cannot be introduced in the legislative council. So, A, B
and C are incorrect.

The Constitution lays down a special procedure for the passing of Money Bills in the state
legislature.

After a Money Bill is passed by the legislative assembly, it is transmitted to the legislative council
for its consideration. The legislative council has restricted powers with regard to a Money Bill.

It cannot reject or amend a Money Bill. It can only make recommendations and must return the bill
to the legislative assembly within 14 days. The legislative assembly can either accept or reject all or
any of the recommendations of the legislative council.

Q Source: Chapter 29: State Legislature: Indian Polity: M Laxmikanth

88 Consider the following statements.


1. Eucalyptus plantations supply moisture to the soil making them suitable for arid and semi-arid
plantation agriculture.
2. When grown using wastewater, eucalyptus plantations can remove toxic metals.

Which of the above is/are correct?


A. 1 only
B. 2 only
C. Both 1 and 2
D. None

www.insightsias.com 60
Total Marks : 200
TEST - 23 (MOCK TEST - 18) Mark Scored : 0

Your Answer :
Correct Answer : B

Answer Justification :

Justification: It has been suggested that we should plant eucalyptus trees for arresting ecological
degradation.

Justification: Statement 2: Growing eucalyptus along sewage ponds absorbs all surplus
wastewater. It is said that these trees are thirsty of water and guzzle a lot of water.

Statement 2: However, where water is sufficient or there is a lot of waste water, it is utilized for
tree growth which then could yield fuelwood, helping in many ways.

Eucalyptus plantations are known to sequester, tolerate and accumulate high levels of various
heavy metals.

Q Source: Revision: 6th Social Science NCERT

89 Alfatoxins are produced by

A. Virus
B. Protozoa
C. Bacteria
D. Molds fungi

Your Answer :
Correct Answer : D

Answer Justification :

Learning: Aflatoxins are poisonous and cancer-causing chemicals that are produced by certain
molds which grow in soil, decaying vegetation, hay, and grains. A mold is a fungus that grows in the
form of multicellular filaments called hyphae. In contrast, fungi that can adopt a single-celled
growth habit are called yeasts.

They are regularly found in improperly stored staple commodities such as cassava, chili peppers,
corn, cotton seed, millet, peanuts, rice, sorghum, sunflower seeds, tree nuts, wheat, and a variety of
spices.

When contaminated food is processed, aflatoxins enter the general food supply where they have
been found in both pet and human foods, as well as in feedstocks for agricultural animals.

Animals fed contaminated food can pass aflatoxin transformation products into eggs, milk products,
and meat.

Q Source: Revision: Additional Research: CSP 2013

www.insightsias.com 61
Total Marks : 200
TEST - 23 (MOCK TEST - 18) Mark Scored : 0

90 Consider the following about Dhrupad, a genre in Hindustani classical music.


1. It finds mention in Ain-e-Akbari.
2. The phrases of Dhrupad alapa are slow and contemplative in the beginning, but the tempo increases
in stages.
3. It is a sub-set of Khyal form of classical singing.

Select the correct answer using the codes below.


A. 1 and 2 only
B. 2 and 3 only
C. 2 only
D. All of the above

Your Answer :
Correct Answer : A

Answer Justification :

Justification: Abul Fazl, courtier and chronicler at the court of the Emperor Akbar, defines the
dhrupad verse form in his Ain-e-Akbari as "four rhyming lines, each of indefinite prosodic length.
So, 1 is correct.

The elaboration of Dhrupad alap is done using the syllables of a mantric phrase, it is slow in the
beginning and raises tempo after some time. So, 2 is correct.

Dhrupad and khyāl are the two forms of classical singing that exist today in North India. So, 3 is
wrong.

Q Source: Revision: Additional Research: CSP 2012

91 Which of the following are used as leguminous green manuring crops?


1. Jowar
2. Maize
3. Sannhemp
4. Cowpea

Select the correct answer using the codes below.


A. 1, 2 and 3 only
B. 3 and 4 only
C. 1 and 4 only
D. 2, 3 and 4 only

Your Answer :
Correct Answer : B

Answer Justification :

Justification: Abul Fazl, courtier and chronicler at the court of the Emperor Akbar, defines the
dhrupad verse form in his Ain-e-Akbari as "four rhyming lines, each of indefinite prosodic length.
www.insightsias.com 62
Total Marks : 200
TEST - 23 (MOCK TEST - 18) Mark Scored : 0

So, 1 is correct.

The elaboration of Dhrupad alap is done using the syllables of a mantric phrase, it is slow in the
beginning and raises tempo after some time. So, 2 is correct.

Dhrupad and khyāl are the two forms of classical singing that exist today in North India. So, 3 is
wrong.

Q Source: Revision: Additional Research: CSP 2012

92 Which of the following about the tectonic ‘Indian Plate’ is NOT correct?

A. It is a major tectonic plate.


B. It forms a convergent late boundary with the Himalayas.
C. The Indian plate is tectonically separated from the Peninsular India plate.
D. The plate extends to Pakistan and Myanmar as well.

Your Answer :
Correct Answer : C

Answer Justification :

Justification: Plates are divided between major and minor plates based on their geographical
coverage. Indian plate is thus a major plate. So, (a) is correct.

The subduction zone along the Himalayas forms the northern plate boundary in the form of
continent— continent convergence. So, (b) is correct.

Peninsular plate is an integral part of the Indian plate. So, (c) is incorrect.

In the east, it extends through Rakinyoma Mountains of Myanmar towards the island arc along the
Java Trench. The Western margin follows Kirthar Mountain of Pakistan. So, (d) is correct.

Q Source: Revision: Chapter 4: Fundamentals of Physical Geography: 11th NCERT

93 The “Transformative Carbon Asset Facility (TCAF)”, that would help developing countries pay for
emission reductions, was launched by

A. World Bank
B. UNFCCC
C. WWF
D. UNEP

Your Answer :
Correct Answer : A

Answer Justification :

www.insightsias.com 63
Total Marks : 200
TEST - 23 (MOCK TEST - 18) Mark Scored : 0

Learning: The World Bank has launched a $500 million dollars Transformative Carbon Asset
Facility (TCAF) to help developing countries pay for emission reductions and combat climate
change.

It is market-based scheme specially designed to help developing countries and would establish the
world’s first programmatic carbon market.

The scheme will reward countries for reducing emissions by paying a fee for each tonne of carbon
dioxide (CO2) reduced. The facility will help countries implement their Intended Nationally
Determined Contributions (INDCs).

Q Source: https://tcaf.worldbank.org/

94 Consider the following about the State Finance Commission (SFCs).


1. It is constituted every five years by the Union Finance Commission (UFC).
2. It reviews the financial position of the Panchayats and Municipalities and recommends distribution
of resources from state exchequer.
3. The Governor is required to cause every recommendation made by the SFC with an explanatory
memorandum, as to the action taken thereon, before the Legislature of the State.

Select the correct answer using the codes below.


A. 1 and 2 only
B. 2 and 3 only
C. 1 and 3 only
D. 1, 2 and 3

Your Answer :
Correct Answer : C

Answer Justification :

Justification: Statement 1: The governor of a state, after every five years, constitutes a finance
commission to review the financial position of the Panchayats. It is not the UFC that constitutes it.

It makes recommendations to the Governor as to the principles which should govern

The distribution between the State and the Panchayats of the net proceeds of the taxes,
duties, tolls and fees leviable by the State, which may be divided between them under this
Part and the allocation between the Panchayats at all levels of their respective shares of such
proceeds;

The determination of the taxes, duties, tolls and fees which may be assigned as, or
appropriated by, the Panchayats;

The grants-in-aid to the Panchayats from the Consolidated Fund of the State;

www.insightsias.com 64
Total Marks : 200
TEST - 23 (MOCK TEST - 18) Mark Scored : 0

The measures needed to improve the financial position of the Panchayats;

Q Source: Chapter 34: Panchayati Raj: Indian Polity: M Laxmikanth

95 Electrocardiography is based on the fact that

A. The heart shows electric activity which can be recorded


B. Beating of the heart generated microwaves that can be read by an instrument
C. Heart is connected to the brain which generates noticeable magnetic waves that can be
graphed
D. The biological energy drawn by the heart from the body can be mapped on a device

Your Answer :
Correct Answer : A

Answer Justification :

Learning: The electrocardiogram (ECG or EKG) is a diagnostic tool that is routinely used to assess
the electrical and muscular functions of the heart.

The heart is a two stage electrical pump and the heart's electrical activity can be measured by
electrodes placed on the skin.

Like the spark-plug of an automobile the heart generates a number of "sparks" per minute. Each
"spark" travels across a specialized electrical pathway and stimulates the muscle walls of the heart
to contract and expand in a certain way.

Q Source: Revision: 11th Biology NCERT

96 Which of the following can be best characterized as a “biome”?

A. Tundra region
B. Lake Baikal
C. Western Himalayan forests
D. Ganges River

Your Answer :
Correct Answer : A

Answer Justification :

Justification: Biome is a community of flora and fauna that covers a large geographical area.

In the diagram below, several levels of ecosystem have been given.

www.insightsias.com 65
Total Marks : 200
TEST - 23 (MOCK TEST - 18) Mark Scored : 0

A biome may or may not have a collection of endemic species. It is not a meeting boundary, and
distinct from small ecosystems or communities.

Some of the major biomes of the world are: forest, grassland, desert and tundra biomes.

Q Source: Revision: Fundamentals of Physical Geography: 11th NCERT

97 Consider the following statements.


1. The Chief Election Commissioner and the two Election Commissioners draw salaries and allowances
at par with those of the Judges of the Supreme Court of India.
2. The Chief Election Commissioner or an Election Commissioner holds office for a term of four years
from the date on which he assumes his office.
3. The Cabinet Committee on Political Affairs allocates the business amongst the Chief Election
Commissioner and their Election Commissioners.

Select the correct answer using the codes below.


A. 1 only
B. 2 and 3 only
C. 1 and 2 only
D. 1 and 3 only

Your Answer :
Correct Answer : A

www.insightsias.com 66
Total Marks : 200
TEST - 23 (MOCK TEST - 18) Mark Scored : 0

Answer Justification :

Justification: Statement 1: They have similar privileges and perks as that of a SC judge of India.

Statement 2: It is six years.

Statement 3: The Election Commission may be by unanimous decision, regulate the procedure for
transaction of to business as also allocation of its business amongst the Chief Election
Commissioner and their Election Commissioners.

Q Source: Chapter on Supreme Court: Indian Polity: M Laxmikanth

98 Nitrogen dioxide in excess in air is harmful to


1. Photosynthesis in plants
2. Textile fibres
3. Children due to risk of acute respiratory diseases

Select the correct answer using the codes below.


A. 1 only
B. 2 and 3 only
C. 1 and 3 only
D. 1, 2 and 3

Your Answer :
Correct Answer : D

Answer Justification :

Justification: Higher concentrations of NO2 damage the leaves of plants and retard the rate of
photosynthesis.

The irritant red haze in the traffic and congested places is due to oxides of nitrogen.

Nitrogen dioxide is a lung irritant that can lead to an acute respiratory disease in children. It is
toxic to living tissues also.

Nitrogen dioxide is also harmful to various textile fibres as it leads to breaking of these fibres.

Q Source: Revision: Unit 14: 11th Chemistry NCERT

99 Consider the following statements.


Assertion (A): Before independence, local self-government never received statutory recognition.
Reason (R): Provinces were never given the authority to legislate for local self-government in pre-
independent India.

In the context of the above, which of these is correct?


A. A is correct, and R is an appropriate explanation of A.
B. A is correct, but R is not an appropriate explanation of A.

www.insightsias.com 67
Total Marks : 200
TEST - 23 (MOCK TEST - 18) Mark Scored : 0

C. A is correct, but R is incorrect.


D. Both A and R are incorrect.

Your Answer :
Correct Answer : D

Answer Justification :

Justification: Under the provincial autonomy scheme introduced by the Government of India Act of
1935, local self-government was declared a provincial subject.

Under the dyarchical scheme introduced in Provinces by the Government of India Act of 1919, local
self-government became a transferred subject under the charge of a responsible Indian minister.
So, both A and R are wrong.

One of the reasons for its late constitutional recognition was the lack of political consensus in India,
especially between the Centre and States.

Q Source: Chapter 35: Municipalities: Indian Polity: M Laxmikanth

100 Rh grouping in human blood should be checked


1. When donating blood to someone
2. During the delivery of a child

Which of the above is/are correct?


A. 1 only
B. 2 only
C. Both 1 and 2
D. None

Your Answer :
Correct Answer : C

Answer Justification :

Justification: An Rh-ve person, if exposed to Rh+ve blood, will form specific antibodies against the
Rh antigens. Therefore, Rh group should also be matched before transfusions. So, (c) is related.

During the delivery of the first child, there is a possibility of exposure of the maternal blood (if Rh
–ve) to small amounts of the Rh+ve blood from the foetus. So, (a) is correct.

An injury or a trauma stimulates the platelets in the blood to release certain factors which activate
the mechanism of coagulation. Calcium ions play a very important role in clotting.

Q Source: Revision: 11th Biology NCERT

www.insightsias.com 68

You might also like